Hurst review

Lakukan tugas rumah & ujian kamu dengan baik sekarang menggunakan Quizwiz!

The client has been prescribed 0.6 units of insulin/kg /day. The client weighs 214 pounds (97 kg). What is the amount of insulin the client can receive in a day? (Round to the nearest whole number)

Answer: 58 units per day 97 kg x 0.6 units = 58.2 units = 58 units The average adult dose of insulin is 0.4-1.0 units/kg/day. Rounding Rules for Whole Units: 0.1 -0.4 = round down to whole unit 0.5-0.9 = round up to whole unit

A client admitted to a psychiatric facility is refusing all medications. The nurse notes the client appears to be responding to auditory hallucinations. What actions by the nurse would be appropriate? 1. Assign staff to stay with client. 2. Place client into a seclusion room. 3. Ask client to explain auditory sounds. 4. Frequently reorient client to reality. 5. Turn up radio to mask hallucinations.

1 and 4. CORRECT. Auditory hallucinations, also called "paracusia", are extremely frightening. The client's intense fear may result in striking out at staff, visitor or other clients, and can even cause the client to do self-harm. Nurses must focus on safety by remaining with the client at all times in a quiet room. Reinforcing that feeling of being safe while frequently reorienting the client to reality are priority actions that may continue for hours until the client becomes calmer. 2. INCORRECT. Though the idea of a quiet environment with decreased stimuli is important, a frightened client with hallucinations should never be left alone. 3. INCORRECT. Asking a client to explain hallucinations feeds into the delusion. If the client indicates hearing messages or commands being given, the nurse would acknowledge the hallucination may seem real to the client, but is not perceived by anyone else. 5. INCORRECT. The best approach for a hallucinating client is to decrease stimuli and move client to a quiet environment. Chaos or loud noises would not mask hallucinations perceived within the client's mind.

The charge nurse on a surgical unit is assigning tasks to an LPN float nurse. What client(s) would be most appropriate for the LPN? 1. Client with Gardner-Wells tongs scheduled for pin care. 2. Client needing straight-cathed for a stat urine sample. 3. Client to be ambulated following open cholecystectomy. 4. Client requiring sterile dressing change to left wrist burn. 5. Client with total laryngectomy due for trach care this shift.

1, 2 and 4. CORRECT. An LPN can perform tasks on a stable client, as long as the task does not require any assessment of that client. The LPN can perform sterile pin care on stable clients with any form of long-term traction such as Gardner-Wells tongs. Obtaining a urine sample by straight-cathing a client is also within the LPN's scope of practice. Sterile dressing changes on burns can be performed by an LPN, though the RN should indicate any need to view or assess the wound before the LPN redresses the site. 3. INCORRECT. A client needing post-operative ambulation could indeed be accompanied by an LPN. However, despite the open cholecystectomy, the client could also be ambulated by a UAP so the LPN can be utilized for a more complex task. 5. INCORRECT. Tracheostomy care must be completed by an RN because of the potential for tracheal suctioning, as well as the need to assess lung sounds before and after the procedure.

An RN has delegated several tasks to be completed before end of shift. What delegation "right" was violated by the nurse? 1. LPN asked to obtain stat vital signs on client reporting chest pain. 2. UAP asked to remove foley catheter prior to client discharge. 3. UAP assigned to empty and measure fluid in client urinal. 4. LPN instructed to check drainage in client's closed drainage unit (CDU) for amount. 5. UAP ordered to obtain finger-stick blood sugar on client at four pm.

1, 2, 4, and 5. CORRECT. The rights of delegation include the right task, right circumstances, right person and the right direction followed by the right supervision or evaluation. The RN has made several errors in utilizing correct delegation principles. A client reporting chest pain is considered unstable and should not be assigned to the LPN, even for vitals. Removal of invasive lines cannot be assigned to a UAP, which includes removal of a foley catheter. Chest tube drainage should be evaluated by an RN in order to assess color, consistency and amount during the shift. Blood glucose levels obtained by Accu-check, also referred to as finger stick, can not be performed by a UAP and reported to the LPN or RN. 3. INCORRECT. The UAP is qualified to empty a urinal or bedpan as part of assisting clients with ADL's and can also measure the amount of fluid at that time. Both task and individual meet the correct "rights" of delegation.

Following chemotherapy for acute lymphocytic leukemia (ALL), the client's lab results indicate a white blood count of 1000 cells mm3. What measures should the nurse institute immediately? 1. Request to change IM antiemetic medication to oral pill. 2. Have client increase fresh fruits and vegetables in diet. 3. Obtain client's temperature at least every two hours. 4. Move client into isolation with a negative flow room. 5. Remove fresh flowers and limit visits from children.

1, 3 and 5. CORRECT. The client has neutropenia, an extreme decrease in the neutrophils of white blood cells. As the main infection fighting faction of WBCs, the client will be at great risk for infection. Reducing invasive procedures by eliminating an intramuscular injection is an important and positive change. Fever is generally an early sign of infection, so taking the client's temperature frequently may alert staff to problems before a serious complication occurs. Fresh flowers contain a variety of bacteria that could be deadly to this client while children are often carriers of viruses without actually showing indications of illness. 2. INCORRECT. Actually, clients with neutropenia are instructed to avoid fresh or raw fruits and vegetables since even proper handling and cleaning can leave traces of bacteria behind. 4. INCORRECT. Although the client should be placed in a private room with a closed door, negative airflow is not necessary. This type of room is reserved for those with active tuberculosis.

A client with deep partial thickness burns to arms and legs is admitted to the burn unit. The nurse knows elevated results are most likely to be noted initially in what laboratory tests? 1. Hematocrit 2. Albumin 3. Potassium 4. Creatinine 5. Magnesium

1, 3, & 4. CORRECT. The physiology of the body changes significantly following a major burn. Hematocrit increases as the fluid from the vascular spaces leaks into the interstitial tissues. Because of lysis of cells, potassium is released into the circulation, leading to hyperkalemia. The kidneys are impacted by the decreased cardiac output as well as the myoglobin released by the lysed cells. This causes creatinine to become elevated. 2. INCORRECT. Albumin, a body protein, is lost through the damaged skin areas and secondary to increased capillary permeability. 5. INCORRECT. Magnesium is a major electrolyte necessary for both muscle and nerve function. Since the body does not produce magnesium naturally, humans need a well-balanced diet which includes a variety of vegetables and seeds. Levels of magnesium are not affected during the initial period after a burn.

The nurse is educating a client newly diagnosed with chronic stable angina about Nitroglycerin SL. What points should the nurse include? 1. Nitroglycerin increased blood flow to the heart. 2. Take one nitroglycerin every five minutes until pain stops. 3. Sit or lie down when taking nitroglycerin. 4. The most common side effect is a headache. 5. Keep nitroglycerin in a clear, plastic bottle.

1, 3, & 4. Correct: Nitroglycerin dilates the coronary arteries to allow more oxygen to get to the heart muscle. Because nitroglycerin also dilates all arteries and veins, the client's BP will drop. So they could faint. To prevent this, they should sit or lie down when taking the nitro. The most common side effect is that the client will get a headache. It is not life threatening, but advise the client that this will occur. 2. Incorrect: One Nitroglycerin can be taken SL every five minutes up to three doses. If pain is not relieved, the EMS should be activated. The client may be having an MI rather than angina. 5. Incorrect: Nitroglycerin should be stored in a dark, glass bottle so that it does not lose its potency.

A nurse is hired to address quality improvement at a long-term care facility. The nurse quickly notes several legal issues requiring intervention and correction. What situation noted by the nurse represents the greatest legal or ethical risk in the facility? 1. A LPN regularly leaves pills in client's room for client to take after meals. 2. The UAP refills the water pitcher for a client on fluid restrictions. 3. Staff asks family members to assist in turning an obese client. 4. The UAP loosens trach ties at bath-time to apply cream on neck.

1. CORRECT. All the situations have the potential to become problematic. However, leaving medications unattended in a client's room violates multiple protocols as well as legal issues. The nurse must witness the client actually ingesting medication to legally sign off the drug as 'given'. Also the nurse should observe the client in case of accidental choking. Unattended pills could easily be ingested by someone other than the designated individual, or the client may simply throw the medication away. 2. INCORRECT. All staff should have been clearly instructed regarding the client's fluid restrictions. However, filling the pitcher does not necessarily mean the client drank the extra fluid. The nurse is responsible to instruct staff on any type of client restrictions. A sign noting fluid restrictions could be placed above the bed without violating HIPAA regulations as long as no name appears on the sign. 3. INCORRECT. Regardless of whether a client is obese, family should never be asked to assist staff in moving or positioning an individual. Family is neither properly trained nor covered by facility insurance for personal injury. Even if family offers to assist, the staff should utilize only other staff members. 4. INCORRECT. The UAP is never permitted to untie or loosen trach ties for any reason. If the client needs, or requests, cream to the neck area, the RN can do so during trach care. If the back of the client's neck is irritated, the RN can also address that issue with proper interventions.

When giving report to the on-coming staff, the night nurse reports a chaotic shift with short-staffing. Later, the day nurse finds multiple uncompleted tasks. What task is most concerning to the charge nurse? 1. A dose of narcotic medication not signed off. 2. A prescribed wound specimen not collected. 3. Primary healthcare provider not notified of abnormal lab results. 4. A finger stick blood sugar not obtained.

1. CORRECT. There are multiple concerns when any medication is not properly recorded, and particularly a narcotic medication. Because of the legalities involved with Schedule II or III drugs, narcotics are tracked and accounted for by two nurses. In this instance, the drug was not acknowledged, making it difficult to determine when, or even if, the client received the medication. 2. INCORRECT. Not obtaining or sending a prescribed wound specimen may delay possible treatment by an extra day, based on how soon the nurse completes this task. However, it is possible the primary healthcare provider may compensate for this issue by ultimately changing the medication or dose ordered when lab results arrive. 3. INCORRECT. While this has the potential to be serious, there is not enough data provided. Abnormal results could be life-threatening or minimal enough to require no intervention. No determination can be made. 4. INCORRECT. The primary healthcare provider will be notified of the missed blood glucose level. The action determined will depend on whether a glucose level has been obtained since the missed level, or if the primary healthcare provider prescribes a stat level now. However, there is a more serious issue.

What action should the nurse take when caring for a client who has a subarachnoid screw? 1. Keep connections tight. 2. Use clean technique when caring for screw. 3. Clean daily with hydrogen peroxide. 4. Maintain a wet to dry dressing around site.

1. Correct. In order to prevent infection, all connections should be tight. We do not want leaking of the CSF. 2. Incorrect. Aseptic technique should be used when cleaning the screw insertion site in order to prevent infection. 3. Incorrect. Sterile normal saline can be used to clean the area around the site. 4. Incorrect. Dressings should be dry and intact. Wet to dry dressings are used as a method of debridement.

A mom two days post delivery reports abdominal pain while breast feeding her newborn. What should the nurse tell the client? 1. "You are getting a surge of oxytocin when you breast feed." 2. "I need to let the doctor know that you need a pelvic exam." 3. "Lie down so I can massasge your fundus." 4. "You cannot have pain medication while you are breast feeding."

1. Correct: Afterpains are common for the first 2 to 3 days and will continue to be common if the mother chooses to breast feed. Every time she nurses the baby, she is going to get a surge of oxytocin, which makes the uterus contract. 2. Incorrect: A pelvic exam is not necessary. Afterpains are common after delivery for 2-3 days. Longer if mom is breastfeeding. 3. Incorrect: You do want to massage the fundus and check fundal hight while the client is in the hospital. However, the clues in the question indicate a surge of oxytocin while breastfeeding. This is normal, so you can relieve mom's anxiety. 4. Incorrect: The afterpains can be severe, so mom's may receive pain medication while breastfeeding.

A client with chronic liver disease has ascites and is being treated with an albumin infusion. What should the nurse anticipate and monitor in this client? 1. Fluid volume excess 2. Cellular edema 3. Severe hypotension 4. Decreasing CVP

1. Correct: Albumin is a hypertonic solution. This type of solution will draw fluid from the cell into the vascular space. This builds up the volume in the vascular space. Therefore, the nurse must watch for fluid volume excess. Hypertonic solutions are used in clients who have 3rd spacing, severe edema, or ascites. 2. Incorrect: Since hypertonic solutions, such as albumin, pull fluid from the cell into the vascular space, we would worry about cellular dehydration and shrinkage, not cellular edema. 3. Incorrect: As the fluid is pulled from the cells into the vascular space, you would expect to see an increase in the BP as the volume in the vascular space increases. You know... more volume, more pressure! We would be watching for hypertension, not hypotension. 4. Incorrect: Think about what we said about the BP when considering the CVP. Since the volume in the vascular space increases with hypertonic solutions, you would also expect the CVP to increase as well. We have to watch closely to make sure that we do not start seeing signs that we are overloading the heart when administering hypertonic solutions. So we will watch this client carefully for an increasing CVP.

A client who had a cerebral vascular accident (CVA) is now having Cheyne-Stokes respirations ranging from 12-30 breaths/minute. BP 158/108, HR 46. Based on this assessment, which acid/base imbalance does the nurse anticipate that this client will develop? 1. Respiratory acidosis 2. Respiratory alkalosis 3. Metabolic acidosis 4. Metabolic alkalosis

1. Correct: Causes of respiratory acidosis include any causes of decreased respiratory drive, such as drugs (narcotics) or central nervous system disorders. With a massive cerebral vascular accident (CVA or stroke), the respiratory center in the brain is impaired and affects oxygenation. Cheyne-Stokes respirations are characterized by progressively deeper and sometimes faster respirations followed by periods of apnea. This leads to acidosis and often times respiratory arrest. 2. Incorrect: Respiratory alkalosis includes hyperventilation and tachypnea which does not describe the characteristics of Cheyne-Stokes respirations. 3. Incorrect: Compensation for metabolic acidosis caused by disorders like DKA includes tachypnea with deep respirations called Kussmaul's respirations. Here, we have a respiratory problem, not a problem that started with a metabolic issue. 4. Incorrect: The most common cause of metabolic alkalosis is vomiting, and this is clearly a respiratory problem, not metabolic.

A 7 month old infant is brought to the emergency department with a sudden onset of inconsolable crying and currant jelly-like stools. The infant is drawing up the knees toward the abdomen and grimacing. What diagnosis should the nurse anticipate? 1. Intussusception 2. Hirschsprung's Disease 3. Pyloric Stenosis 4. Meconium Ileus

1. Correct: Intussusception is a condition in which a piece of the bowel telescopes in on itself, forming an obstruction. This causes a sudden onset of cramping and abdominal pain. The client tends to be inconsolable and draws the knees upward in response to the pain. The stool may appear normal at first and then currant jelly-like stools may be noticed as blood and mucus become mixed with the stool. 2. Incorrect: Hirschsprung's disease, known as aganglionic megacolon is a congenital anomaly in which there is an absence of nerves in a portion of the bowel, typically the sigmoid colon. This results in mechanical obstruction. Here, you would see constipation and abdominal distention. If stools are passed, they are often ribbon-like that have a foul smell. 3. Incorrect: Pyloric Stenosis is a condition in which there is enlargement of the pylorus. Symptoms include projectile vomiting due to the pressure that increases in the stomach as a result of the inability of the food to pass through the enlarged pylorus to the small intestine. 4. Incorrect: Meconium ileus is a bowel obstruction that results when the first infant stools (meconium) are thicker and stickier than normal. This blockage typically occurs in the ileum of the small intestines, and the cause of most cases of a meconium ileus in infants is cystic fibrosis.

A 7 month old is being evaluated in the emergency department for a possible head injury following a reported fall from the parent's bed. What would the nurse consider when evaluating the fontanels for evidence of increased intracranial pressure? 1. The anterior fontanel should be open at 7 months of age. 2. The anterior fontanel closes at 2 to 4 months of age. 3. The posterior fontanel should be open at 7 months of age. 4. The posterior fontanel closes at 4 to 6 months of age.

1. Correct: The anterior fontanel closes between 12 to 18 months of age. The nurse could assess the anterior fontanel in this 7 month old with a normal finding being soft and flat. A bulging anterior fontanel would be indicative of increased intracranial pressure. 2. Incorrect: The anterior fontanel does not close until 12 to 18 months of age. Closure before this time would be considered premature closure which could affect brain growth. 3. Incorrect: The posterior fontanel should be closed in this client. The posterior fontanel is expected to close at 2 to 3 months of age. Therefore, the posterior fontanel would not be useful for assessing for increased intracranial pressure in this 7 month old. 4. Incorrect: The posterior fontanel closes at 2 to 3 months of age, not 4 to 6 months, and would be expected to be closed in this 7 month old infant.

A client is admitted to the cardiac floor in heart failure. The lung sounds reveal crackles bilaterally, and the BP is 160/98. The client has been on diuretics at home and the potassium level is 3.3 mEq/L (3.3 mmol/L). Which diuretic would the nurse anticipate being prescribed for this client to minimize potassium loss? 1. Spironolactone 2. Furosemide 3. Bumetanide 4. Hydrochlorothiazide

1. Correct: The client's potassium level is low. Spironolactone is a potassium sparing diuretic which would cause the potassium to be retained. 2. Incorrect: Furosemide is a potassium depleting diuretic which would further deplete the potassium level. 3. Incorrect: Bumetanide is a potassium depleting diuretic which would further deplete the potassium level. 4. Incorrect: Hydrochlorothiazide also leads to potassium loss, which would further deplete the potassium level.

A client is experiencing a panic attack. What priority action should the nurse take? 1. Instruct client to deep breathe with the nurse. 2. Teach relaxation techniques. 3. Inform client that symptoms will be gone in 20-30 minutes. 4. Hold the client gently for 5 minutes.

1. Correct: The most important action for the nurse to take is to slow down the client's breathing so that they do not end up in respiratory alkalosis from hyperventilation. 2. Incorrect: Now is not the time to teach! They cannot concentrate on anything but the panic they feel. 3. Incorrect: Again teach them this when they are not having a panic attack. Also teach ways to stop the anxiety from escallating. 4. Incorrect: Approach the client in a nonthreatening manner. Give them space. Do not add to the anxiety by getting in their space.

Which medication prescription should the nurse question for a client diagnosed with nephrotic syndrome? 1. Ibuprofen 2. Enalapril 3. Prednisone 4. Cyclophosphamide

1. Correct: The prescription of ibuprofen, a NSAID, would be questioned. Ibuprofen is a nonsteroidal ant-inflammatory medication. NSAIDs can cause acute interstitial nephritis and acute tubular nephritis. The client with nephrotic syndrome currently has damage to the micro blood vessels in the kidneys. 2. Incorrect: Enalapril, an angiotensin converting enzyme inhibitor (ACE), is prescribed for nephrotic syndrome to decrease the intraglomerular pressure. The inhibition of the angiotensin converting enzyme (ACE) results in a reduction of proteinuria. Also an ACE inhibitor blocks aldosterone secretion. This prescription is appropriate. 3. Incorrect: Prednisone's classification is a corticosteroid. Client's with nephrotic syndrome leak protein from the blood into urine. Prednisone action is to reduce the inflammation of the kidneys, and results in decrease proteinuria. The prescription of a corticosteroid is applicable 4. Incorrect: Cyclophosphamide's pharmacology classification is an alkylating agent. This medication is prescribed for the treatment of nephrotic syndrome to suppress the body's immune system. The prescription of cyclophosphamide is appropriate for this client.

A client who has terminal cancer tells the nurse that the opioid prescription, which is at the highest recommended dose, is not relieving the pain. What should the nurse tell the client? 1. "I will ask your primary healthcare provider to increase your dose of medication." 2. "You cannot have a higher dose of pain medication since you are at the maximum dose." 3. "Opioid addiction is a major concern. You don't want to take too much of this medication." 4. "Let's try some lemon essential oil to decrease your pain level."

1. Correct: There is no ceiling on the dose of an opioid for a cancer client. Dosage is only limited by side effects. It is client dependent, so this is an appropriate response by the nurse. 2. Incorrect: This statement is not true for the cancer client and would be an inappropriate response by the nurse. 3. Incorrect: This client is dying. We are not worried about addiction. We are worried about easing the pain and helping the client die with as little to no pain as possible. 4. Incorrect: Lemon essential oil is helpful in decreasing nausea. It is not useful in relieving pain. At this stage in the client's illness, opioid medication is the "gold standard".

A client, admitted to the surgical unit post left thoracotomy, is drowsy. Vital signs on admit are T 99.8ºF (37.6ºC), HR 94, R 16/shallow, BP 100/68. ABGs are pH 7.33, PCO2 48, HCO3 24. What action should the nurse initiate? 1. Have client take deep breaths. 2. Administer naloxone. 3. Tell the client to breathe faster. 4. Medicate for pain.

1. Correct: This client had chest surgery and the pCO2 is high. What are you worried about? Hypoventilation. Yes, the client is probably hurting due to the incision and does not want to take deep breaths. In order to get rid of the excess CO2 the client needs to turn, cough, and deep breathe. Incentive spirometry can be provided to assist the client with this effort. 2. Incorrect: This client has mild respiratory acidosis after surgery. The nurse can fix this by waking the client up and instructing the client to take deep breaths or have the client use incentive spirometry. 3. Incorrect: Breathing faster will only work for a few minutes. The problem is the client needs to breathe deeper to get more oxygen to the tissue and more CO2 out of the lungs. Hyperventilating will lead to respiratory alkalosis. 4. Incorrect: No more sedation! The client is not breathing enough. This client needs to take deep breaths.

A client in her third trimester comes to the clinic for a routine prenatal visit. The nurse notes a weight gain of 4 pounds (1.8 kg) in a week. What action should the nurse take? 1. Check urine for protein. 2. Educate on proper weight gain during pregnancy. 3. Notify the primary healthcare provider. 4. Send client to the labor and delivery unit.

1. Correct: We are worried about pre-eclampsia, so we need to check the client's BP and check urine for protein. 2. Incorrect: If the client's BP and urine are ok, then you can educate the client about proper weight gain. 3. Incorrect: It is a little premature to notify the primary healthcare provider. Let's check some things first. 4. Incorrect: Again, let's check the client out some more first.

Which interventions would the nurse implement for a client with a right total hip arthroplasty performed 6 hours ago? 1. Remove the abductor pillow. 2. Place a pillow under both knees. 3. Position the feet with the toes pointed upward. 4. Assess client's popliteal, dorsalis pedis, and posterior tibial pulses. 5. Report to the healthcare provider the 15g/dL (9.31mmol/L) Hemoglobin.

1. Incorrect: The abduction pillow is not removed within 6 hours of a total hip arthroplasty. This is an inappropriate intervention. The abduction pillow is attached to the legs to prevent adduction of the hips to decrease the risk of a dislocation of the surgical hip. 2. Incorrect: The nurse should not place a pillow under either knee. The pillows would decrease the circulation to the lower extremities and increase the risk factor for deep vein thrombosis (DVT). Also, an abduction pillow is attached to the lower extremities. 5. Incorrect: The normal hemoglobin range for a male client is 14-15 g/dL (8.7-11.2 mmol/L). The client's hemoglobin level is 15g/dL (9.31mmol/L). Since the client's hemoglobin level is within normal range, the nurse will not notify the primary healthcare provider.

The nurse is preparing a class on cancer prevention. Which risk factor should the nurse discuss with the class as being a preventable risk factor? 1. Smoking tobacco 2. Drinking alcohol 3. Eating a high fiber diet 4. Increasing fish consumption 5. Protect skin from sunlight by using tanning beds

1., & 2. Correct: Tobacco is the #1 cause of preventable cancer. Alcohol plus tobacco are co-carcinogenic. 3. Incorrect: A low fiber diet is bad. You don't have much motility in your intestines, so you are retaining carcinogens longer. 4. Incorrect: Increasing fish consumption is a good thing. You want to avoid increased red meat consumption and animal fat. 5. Incorrect: Tanning beds are just as bad as exposure to sunlight. Both cause exposure to ultra-violet radiation.

The nurse is teaching a client diagnosed with mastitis about treatment. The client states she wants to continue breast feeding. What interventions should the nurse include? 1. Get plenty of bed rest. 2. Wear a support bra. 3. Place chilled cabbage leaves on breasts. 4. Take antibiotic prior to breast feeding. 5. Offer the unaffected breast first at each feeding. 6. Take cool showers to relieve breast discomfort.

1., & 2. Correct: Treatment for mastitis includes bed rest. And of course they're going to want a support bra. Because they are going to be engorged. 3. Incorrect: Binding the breast and the use of cabbage leaves will relieve engorgement, but they are only used if breastfeeding is being discontinues permanently. 4. Incorrect: The antibiotic should be taken right after feeding the baby so that the baby does not get much of the antibiotic. 5. Incorrect: If mom is nursing the baby and has mastitis, she should always offer the affected breast first. This will ensure that all the milk is emptied from the affected breast. 6. Incorrect: Mom should take hot showers. This will cause the breast to leak which will help relieve pressure and soften the breast.

The nurse has been assigned to a client with a Steinman pin insertion 48 hours ago. Which pin site care interventions would the nurse implement? 1. Perform pin care daily. 2. Rinse pins with water. 3. Clean with chlorhexidine. 4. Dry the area with clean gauze. 5. Monitor pin site every 10 hours.

1., & 3. Correct: Pin care is prescribed 48 to 72 hours after insertion. The pin care is initiated once a day. Chlorhexidine is prescribed to clean the pin insertion site. 2. Incorrect: The pins are rinsed with sterile saline and not water. 4. Incorrect: The area around the pin site is dried with sterile gauze. The use of clean gauze is not appropriate. 5. Incorrect: The pin site is assessed. Every 10 hours is not often enough to monitor for infection.

The nurse is performing a neurological assessment on an adult client suspected of having a traumatic brain injury (TBI). Which signs/symptoms would indicate to the nurse that the client's ICP is increasing. 1. Projectile vomiting 2. Narrowing pulse pressure 3. Delay in verbal response 4. DTR: left 2+/4+, right 2+/4+ 5. (-) Babinski 6. Glasgow Coma Scale Score 13

1., & 3. Correct: Projectile vomiting can occur because the vomiting center in the brain is being stimulated. Anytime you have a head thing and the client begins to vomit, you have to assume that the ICP is going up! With increasing ICP the client's speech may change - it may become slow or slurred. There is a delay in verbal suggestion. In other words, they may be slow to respond to commands. 2. Incorrect: With increasing ICP the client will develop systolic hypertension with a widening pulse pressure. A narrowed pulse pressure is seen with cardiac tamponade. 4. Incorrect: 2+/4+ is normal (active or expected response). 5. Incorrect: We do not want to see a (+) Babinski in the adult. So a (-) Babinski is a good thing. 6. Incorrect: The best possible score is 15 on the Glasgow Coma Scale. We like to see a high number, like 13-15.

Which selection by the client indicates to the nurse that the client understands food allowed during a vanillylmandelic acid (VMA) test? 1. Milk 2. Caffeine 3. Citrus fruit 4. Chicken 5. Vanilla ice cream

1., & 4. Correct: Milk intake will not alter the production of epinephrine or norepinephrine. The client can drink milk prior to a VMA test. The period prior to a VMA test which measures the amount of production of epinephrine and norepinephrine would not require the client to eliminate chicken. Eating chicken would not alter the production of epinephrine or norepinephrine. 2. Incorrect: The client should not eat/consume caffeine as it will alter the test. 3. Incorrect: Citrus fruit and fruit juices will alter the accuracy of the VMA test which measures the amount of production of epinephrine and norepinephrine. 5. Incorrect: Vanilla ice cream contains vanilla, which can alter the vanillylmandelic acid (VMA) test.

The nurse is educating a group of college students about cancer prevention and screening. Which secondary prevention actions should the nurse include? 1. Annual mamogram starting at age 45. 2. Maintain normal body weight. 3. Cancer support group. 4. Colonoscopy beginning at age 45. 5. Limit or eliminate alcohol intake.

1., & 4. Correct: Secondary prevention includes screenings to pick up on cancer early. Screening is very important because then we have a greater chance for cure or control. Annual mamogram starting at age 45 with two views of each breast is recommended if the client has no family history of breast cancer. Colonoscopy at age 45, then every 10 years after that if there has been no problem is also recommended. 2. Incorrect: Maintaining a normal body weight is considered primary prevention (ways to help prevent the actual occurrence of cancer). 3. Incorrect: Support groups and rehabilitation programs are considered tertiary prevention (focuses on the management of long term care for clients with complex treatments for cancer). 5. Incorrect: Limiting or eliminating alcohol intake is considered primary prevention.

How would the nurse interpret this client's Arterial Blood Gas (ABG) results? pH 7.30 PaCO2 55 mm Hg Bicarb 25 mEq/liter PaO2 93 mm Hg SaO2 95% 1. Respiratory acidosis 2. Respiratory alkalosis 3. Metabolic acidosis 4. Metabolic alkalosis 5. Uncompensated 6. Partially compensated 7. Fully compensated

1., & 5. Correct. Now will a pH of 7.30 make the patient have acidosis or alkalosis? Well it's less than 7.35 so that's a low pH, and you know that a low pH is acidosis. So now we know acidosis, but we still must figure out if it is respiratory or metabolic. Look at the PaCO2, it is 55. That's a lot of CO2, and it's greater than the normal range of 35-45, so the PaCO2 must be acidosis. Now the Bicarb is 25, and a Bicarb of 25 is within the normal range of 22-26. To determine whether this is respiratory or metabolic acidosis, we need to match the pH with either the PaCO2 or the HCO3. The pH that we're trying to match is acidosis, so keep in mind you're trying to match the word acidosis with one of the chemicals. Well look at our problem, the CO2 is also acidosis. So, since these two match and they are both acidosis, we can say this is clearly Respiratory Acidosis because the "respiratory" chemical (CO2) is the one that matches the pH. Now, look at the Bicarb level. The bicarb is normal and doesn't match our pH which is acidosis, so, we can just mark it out. There you have it; this is Respiratory Acidosis. So, is there any compensation going on? No, not yet. The bicarb is still within normal limits. These values indicate uncompensated respiratory acidosis. 2. Incorrect. The pH would need to be above 7.45 and the PaCO2 below 35 for the client to have respiratory alkalosis. 3. Incorrect. The pH would need to be below 7.35 and the Bicarb below 22 for the client to have metabolic acidosis. 4. Incorrect. The pH would need to be above 7.45 and the Bicarb above 26 for the client to have metabolic alkalosis. 6. Incorrect. When partial compensation begins, the bicarb level will be above 26 as it goes UP to put more base in the body. 7. Incorrect. Full compensation will occur when the pH comes back to normal.

The nurse has informed a client diagnosed with heart failure about the treatment plan, including prescriptions for an ACE inhibitor and a 2 gm sodium diet. Which statement by the client would indicate to the nurse that the client understands the treatment plan? 1. "I plan to elevate the head of my bed on concrete blocks so I can sleep better." 2. Instead of using salt, I should use a salt substitute to season my food." 3. "It is important that I weigh myself weekly to monitor for weight gain." 4. "I need to eat foods high in potassium while taking an ACE inhibitor." 5. "A low sodium diet will help decrease swelling in my legs."

1., & 5. Correct: Lying flat when a client has heart failure will cause excess fluid, which has pooled in the extremities while up, to move into the thorax and back up into the lungs. This is why the client can breathe better when the head of the bed is elevated. A low sodium diet decreases fluid retention which decreases preload, the amount of fluid entering the right side of the heart. So, yes, a low sodium diet can help decrease dependent edema. 2. Incorrect: Salt substitutes are high in potassium and can be dangerous when taking an ACE inhibitor. ACE inhibitors block aldosterone, which causes the body to lose sodium and water and retain potassium. 3. Incorrect: The client should weigh self daily, not weekly, and report a weight gain of more than 2-3 pounds (1-2 kg). 4. Incorrect: This client needs to eat food low in potassium since ACE inhibitors cause the retention of potassium.

The emergency department nurse is monitoring a client being admitted in diabetic ketoacidosis (DKA). Which arterial blood gas value would be expected? 1. pH 7.32 2. PaCO2 32 3. HCO3 25 4. PaO2 78 5. SaO2 82

1., &2. Correct: In DKA, the client is acidotic. Normal pH is 7.35-7.45. A pH of 7.32 indicates acidosis and will be expected for a client in DKA. Normal PaCO2 is 35-45. Remember CO2 is considered an acid. The client in DKA will have an increased respiratory rate, so the PaCO2 will either be normal or low. This value of 32 is low and is an expected finding as the body is compensating for the acidosis. 3. Incorrect: Normal HCO3 is 22-26. HCO3 is a base. Initially, the acids bind to the bicarb to reduce the acid levels. Therefore, the HCO3 would be less than 22. So, in DKA, the expected initial finding is a low HCO3​. Keep in mind that with acidosis, as the body compensates later, the kidneys will retain bicarb and you will see the bicarb levels increase. 4. Incorrect: Normal PaO2 is 80-100. An expected finding in DKA will be normal or increased PaO2, not decreased. 5. Incorrect: The client in DKA is kussmauling to blow off the CO2 (acid), so the oxygen saturation of blood will be high if there is no respiratory issue. In this question you are not told that there is a respiratory problem, so you would not expect a low oxygen saturation level.

The nurse is caring for a client following a cholecystectomy. The client reports dizziness, sweating and palpitations after eating meals. The nurse would recommend which actions to alleviate these symptoms? 1. Drink between meals. 2. Reduce intake of carbohydrates. 3. Eat small, frequent meals daily. 4. Sit semi-recumbent for meals. 5. Remain upright for one hour after eating. 6. Lie down on left side after eating.

1., 2, 3, 4., & 6. Correct: Clients are instructed to eliminate all fluids during meals. In some cases, clients may also need to eliminate fluids for one hour before and immediately after meals in order to control symptoms and slow the progress of food through the digestive tract. The symptoms described indicate the client is experiencing dumping syndrome, an adverse response following gastric or bariatric surgery. Clients may also experience tachycardia, nausea or cramping with the intake of food due to surgical restructuring of the gastrointestinal tract. Because this will be a lifetime issue, the nurse must teach the client to adjust eating habits and patterns. Reduction of carbohydrates will help decrease the problem since carbohydrates speed through the digestive track too quickly. Eating smaller, more frequent meals in a semi-recumbent position will further slow food through the digestive tract and eliminate most of the uncomfortable symptoms. After eating, the client should lie down on the left side to keep food in the stomach longer. 5. Incorrect: Sitting up after a meal is counterproductive, since this will increase the speed of food through the digestive tract. Therefore, clients are encouraged to lie down on the left side following meals to slow the processing of food.

The nurse is teaching couples in their final weeks of pregnancy about "Kangaroo Care". What points should the nurse include in this session? 1. Trust in the newborn is an emotional and physiologic need fulfilled through "Kangaroo Care". 2. "Kangaroo Care" requires skin to skin contact between the newborn and parent. 3. The newborn is held quietly for an hour at least 4 times a day. 4. "Kangaroo Care" will only be encouraged if your newborn is premature. 5. Research shows that skin to skin bonding stabilizes the newborn's heart rate.

1., 2., & 5. Correct: In the infant, trust is an emotional need and a physiologic need. So that makes it a priority. Maslow says that "physiologic needs" come first! Skin to skin contact provides trust through physiologic bonding. Baby is placed skin to skin on mom or dad's chest. Research shows that skin to skin holding stabilizes the infant's heart rate, improves O2 sats, regulates the infant's temp., and conserves the infant's calories. They have even found that the breast can change in temperature to warm or cool the infant. 3. Incorrect: The infant is wrapped inside the parent's shirt or covered with a blanket and the baby is quietly held for an hour at least 4 times a week. 4. Incorrect: The focus of "Kangaroo Care" has been the premature baby, but most hospitals now have this as standard care for all babies.

What signs/symptoms would lead the nurse to suspect that a client diagnosed with cirrhosis may be developing hepatic coma? 1. Asterixis 2. Fetor 3. Grey Turner's sign 4. Hyperactive reflexes 5. Squiggly handwriting

1., 2., & 5. Correct: Signs and symptoms that a client diagnosed with cirrhosis is getting worse and headed for hepatic coma include asterixis, fetor, and handwriting changes. 3. Incorrect: Grey Turner's sign is seen with pancreatitis. 4. Incorrect: With hepatic coma, the client is full of toxins, so reflexes will be decreased.

A client is placed on neutropenic precautions. What interventions should the nurse initiate? 1. Use antimicrobial soap for handwashing. 2. Post neutropenic precautions sign on door. 3. Administer acetaminophen for fever greater than 101 degree F (38.3 degrees C). 4. Administer platelets as prescribed. 5. Vital signs at least every 4 hours.

1., 2., & 5. Correct: We want to use antimicrobial soap to wash hands. Anyone planning to enter the client's room needs to know what to do prior to entering, so a sign with necessary instructions should be placed on the closed door. Vital signs should be done every 4 hours, minimally. If needed, take vital signs more frequently. 3. Incorrect: Don't administer acetamenophen. It can be toxic to the liver. 4. Incorrect: Platelets are not needed for a low white count. They are given when the client is thrombocytopenic.

Which assessment findings would indicate to the nurse that a client may have a fracture? 1. Swelling 2. Deformity 3. Crepitus 4. Discoloration 5. Tenting of skin

1., 2., 3. & 4. Correct: Swelling, deformity, crepitus, and discoloration are signs of a fracture. The swelling is caused by fluids and blood that move into the soft tissues. The leaking of blood from the soft tissue or from the bone will result in a discoloration or bruising at the injury site. The most accurate sign of a broken bone is deformity of the bone. An example would be when a bone is bending in an inappropriate direction. 5. Incorrect: Tenting of the skin is not a sign of a fracture. Tenting is the slow return of skin after the skin has been pinched. If tenting is present, this indicates that the client is possibly dehydrated.

The nurse is performing a neurological assesment on a client who reports frequent headaches. What question(s) should the nurse ask during this assessment? 1. "When did the headaches begin?" 2. "What symptoms accompany the headaches?" 3. "Does anything relieve the headaches?" 4. "Does anything make the headaches worse?" 5. "Are you experiencing depression?"

1., 2., 3., & 4. Correct. These are all questions that are part of a focused neurological assessment. You want to inquire about the client's current condition, onset of symptoms, description of symptoms, and associated factors. 5. Incorrect. Keep questions open-ended to gather more data. This is very specific and not the best way to get overall information. A better question would be: "What current and past illnesses have you experienced?"

Which signs/symptoms does the nurse expect to see in a client diagnosed with schizophrenia? 1. Auditory hallucinations 2. Grandiose delusions 3. Religious preaching all the time. 4. Flat affect 5. Abstract reasoning

1., 2., 3., & 4. Correct: Auditory hallucinations are commonly experienced by the client diagnosed with schizophrenia. Delusions of grandiosity like believing they are a famous person or religious figure is a false fixed belief experienced by the client. If the client is in the acute phase of schizophrenia, the person may be overwhelmed by anxiety and is not able to distinguish thoughts from reality. It is thought that delusions may develop to cope with the anxiety. Religiosity is common. The client may carry a bible all of the time and preach to everyone all of the time. The client may have an inappropriate affect, a flat affect, or a blunted affect. 5. Incorrect: This client has concrete thinking which implies over emphasis on specific details and an impairment in the ability to use abstract concepts. For example, during the nursing history you may ask the client what brought them to the hospital and the answer will be "a cab."

The nurse is implementing cast care instructions for a client with a plaster cast applied 2 hours ago. Which cast care instruction would be included? 1. Rest cast on a soft pillow. 2. Keep the cast uncovered until air dried. 3. Mark the cast if there is breakthrough bleeding. 4. Place ice packs on side of the cast for first 24 hours. 5. Use the palms of hands when moving the cast for first 6 hours.

1., 2., 3., & 4. Correct: Until the cast has dried completely, the cast care instructions are to prevent indentations on the cast, reduce swelling, and evaluate any breakthrough bleeding. 5. Incorrect: To prevent indentations in the plaster cast, the cast should be moved with the palms of hands for first 24 to 72 hours.

What sign/symptom would indictate to the nurse that a client has had an inhalation injury? 1. stridor 2. Swallowing difficulty 3. Singed nasal hair 4. Blisters to upper arms 5. Wheezing

1., 2., 3., & 5. Correct: Substernal/intercostal retraction and stridor are bad signs. Remember you will see difficulty swallowing, singed nasal and facial hair, and wheezing. 4. Incorrect: Blisters found on the oral/pharyngeal mucosa is more likely to indicate a smoke or inhalation injury.

What interventions would the nurse implement for a client diagnosed with nephrotic syndrome? 1. Weigh daily 2. Measure abdominal girth 3. Provide skin care 4. Position in semifowlers 5. Intake and output

1., 2., 3., & 5. Correct: The client with nephrotic syndrome is producing less urine. Due to the decrease in urinary output the client is retaining fluid. The client should be weighed daily, and the girth would be measured to evaluate fluid retention. Edematous skin is prone to skin breakdown, so adequate skin care is necessary. Intake and output is required whenever there is a fluid volume problem. 4. Incorrect: This client with nephrotic syndrome does not require positioning in a semifowlers position. Any position of comfort is acceptable.

What signs/symptoms should the nurse assess for when caring for a client at risk for thrombocytopenia? 1. Conjunctival hemorrhage 2. Petechiae on inside of mouth 3. Purpura 4. Fever 5. Blood oozing from IV site

1., 2., 3., & 5. Correct: The problem is a low platelet count, so we are looking for signs/symptoms of bleeding such as conjunctival hemorrhage, petechiae on the arms, legs or inside the mouth, and ecchymosis or purpura. 4. Incorrect: Thrombocytopenia is a decrease in the number of circulating platelets in the blood. Fever is seen with neutropenia

The nurse is planning care for a client admitted with a diagnosis of Alzheimer's Disease. What interventions should the nurse include? 1. Encourage participation in light exercise. 2. Identify doors with pictures. 3. Monitor food intake. 4. Assign unlicensed assistive personnel to bathe client daily. 5. Reminisce about successful and unsuccessful life events. 6. Weigh weekly.

1., 2., 3., & 6. Correct: It is important to keep the client as active as possible by participating in enjoyable things like light exercise, dancing, singing, simple games, and painting. Identify all doors with pictures or easily identifiable labels. Doors to rooms, closets, and bathrooms are especially important for the client to be able to recognize. Monitor food and liquid intake daily. The client can easily forget to eat and drink. This is one reason the client should be weighed weekly as well. 4. Incorrect: Have the client dress in their own clothes whenever possible and perform their own activities of daily living for as long as possible. This helps to maintain self-esteem. 5. Incorrect: Talk about meaningful things. Help the client focus on a successful life events to increase self-esteem. Talking about unsuccessful life events will not increase self-esteem or be helpful to the client.

A 70 year old female client reports an occasional choking sensation over the past 12 hours. What additional symptoms reported by the client would indicate to the nurse that the client may be having a myocardial infarction? 1. Unusual fatigue. 2. Indigestion. 3. Aching jaw. 4. Feeling faint 5. Pain between the shoulder blades. 6. Left arm paresthesia.

1., 2., 3., 4., & 5. Correct: Look at the hints - elderly, female, choking sensation. Women often present with GI signs and symptoms, epigastric complaints, or pain between the shoulders, aching jaw, or choking sensation. The triad of symptoms: feeling of fullness in the abdomen, unusual fatigue, and an inability to "catch one's breath". Remember that the elderly may just faint or only have SOB. 6. Incorrect: Left arm paresthesia sounds more like a stroke rather than an MI.

A client sustains a high-voltage electrical injury while at work. Which interventions should the occupational health nurse initiate? 1. Assess entry and exit wound. 2. Monitor vital signs. 3. Place on a spine board. 4. Connect to cardiac monitor. 5. Perform the rule of nines. 6. Apply cervical collar to neck.

1., 2., 3., 4., & 6. Correct: You need to understand that high-voltage current of electricity damages the vascular system and the nerves nearby. This alteration in the vascular system can damage vital organs, so we worry about organ failure. Electrical burns have two wounds: an entrance burn wound that is generally small and an exit burn wound that is much larger. The electricity goes throughout the body causing damage, and then exits the body. So look for 2 burn wounds. Remember, vessels, nerves, and organs can be damaged. The nurse needs to monitor vital signs frequently, especially those assessing the respiratory and cardiac systems, since we worry about organ damage. Electricity can damage the heart muscle, so the client is at risk for dysrhythmias within 24 hours following an electrical burn. Put the client on continuous cardiac monitoring during this time. Why place the client on a spine board and put a c-collar on? Contact with electricity can cause muscle contractions strong enough to fracture bones, or vertebrae. The force of the electricity can actually throw the victim forcefully. 5. Incorrect: This statement is false. The rule of nines is not used for electrical burns, but for thermal burns. Most of the damage from electrical burns is internal and cannot be determined by using the rule of nines.

Which signs/symptoms would lead the clinic nurse to suspect that a client may have bacterial meningitis? 1. Nuchal rigidity 2. Photophobia 3. (+) Kernig 4. (-) Brudzinski 5. Fever 102.8 F (39.3 C) 6. Reports headache 9/10

1., 2., 3., 5., & 6. Correct. Signs and Symptoms of meningitis include nuchal rigidity, photophobia, a positive Kernig sign, chills and high fever, and severe headache. 4. Incorrect. The Brudzinski sign would be positive.

The nurse on a medical unit is reviewing the data on a client admitted to a medical unit. Which data supports the diagnosis of glomerulonephritis? 1. Malaise 2. Blood pressure - 16O/92 3. 24 hour urinary output - 960 mL 4. Costovertebral angle tenderness 5. Urine specific gravity of 1.040

1., 2., 4., & 5. Correct: A client with glomerulonephritis is retaining toxins. The result is the client has a overall sense of being ill with possible fatigue and decrease interest in activities. A client with glomerulonephritis is producing less and less urine. Due to the retention of fluid, the client's blood pressure is elevated. Costovertebral angle tenderness (CVAT) is elicited by percussing the flank area of the back over both the kidneys. If pain is present, the client would be assessed for a kidney infection. The normal range of urine specific gravity ranges from 1.010 to 1.030. An elevated urine specific gravity of 1.040 is reflective of highly concentrated urine. The client is not diuresing appropriately. The client's specific gravity of 1.040 supports glomerulonephritis. 3. Incorrect: The normal range for a 24 urinary output is 800-2000 mL. The listed 24 hour urinary output of 960 is within the normal range. Urinary output on a client with glomerulonephritis is less than 800 mL per 24 hours.

What interventions should the nurse provide when caring for a client prescribed oxytocin IV? 1. Label IV bag and IV tubing with oxytocin sticker. 2. Monitor for late decelerations. 3. Position client supine. 4. Piggyback oxytocin at the lowest primary IV site. 5. Provide one on one care.

1., 2., 4., & 5. Correct: Always label both the IV bag with an oxytocin sticker and the IV tubing and ports. Nothing else goes through the tubing and we want it easily identified if it must be stopped. The nurse must monitor for late decelerations while the client is receiving oxytocin. If late decels occur, turn of the oxytocin immediately. Oxytocin is given IVPB at the lowest port so that if it has to be turned off, the client does not get more from the primary IV tubing. Oxytocin is a high risk alert drug. This means there is a high risk of client harm, so never underestimate the problems that can occur with oxytocin administration. The client needs one on one care. 3. Incorrect: The client receiving oxytocin can be placed in any position except flat on their back.

The nurse is planning care for a client admitted with a diagnosis of acute renal injury. What interventions should the nurse include in this plan? 1. Provide meticulous skin care. 2. Reposition every 2 hours. 3. Maintain a high carbohydrate, high protein diet. 4. Provide foods low in phosphate. 5. Monitor intake and output. 6. Give IV medications in smallest volume allowed.

1., 2., 4., 5., & 6. Correct: The leading cause of death from acute renal injury is infection, so meticulous skin care and aseptic technique are critical. Repositioning every 2 hours will help to prevent pressure ulcers. Clients in acute renal injury have high phosphorus levels and low calcium levels (remember that inverse relationship?). So they need foods low in phosphorus. Monitor intake and output. The client cannot handle excess fluid at this time. This is also why all IV meds should be administered in the smallest possible volume allowed. 3. Incorrect: The client does need to be on a high carbohydrate, high fat diet to prevent protein breakdown. However, a low protein diet is needed because the kidneys cannot excrete BUN and creatinine.

What actions would be appropriate for the nurse to take when performing peritoneal dialysis on a client diagnosed with renal injury? 1. Dialysate is warmed to body temperature by allowing it to sit out for a short period of time. 2. The dialysate is infused through the catheter into the stomach. 3. Once infused, dialysate remains for prescribed dwell tiime. 4. Withdraws dialysate using a large piston syringe. 5. Assists client to stand if all the drainage is not removed.

1., 3. Correct: These actions are correct. The dialysate should be warmed to body temperature by allowing it to sit out for a short period of time. The dwell time is the length of time that the dialysate stays in the peritoneal cavity. This allows for toxins to be drawn out of the blood and into the peritoneal cavity for removal. 2. Incorrect: The dialysate is infused through the peritoneal catheter into the peritoneal cavity. 4. Incorrect: Allow the dialysate to drain by gravity for 20-30 minutes. 5. Incorrect: The nurse should turn the client from side to side if all the drainage does not come out of the peritoneum.

What assessment finding by the nurse would support a client diagnosis of basilar skull fracture? 1. (+) Halo test 2. Hyper-reflexia 3. Raccoon eyes 4. Battle's sign 5. Kernig sign

1., 3., & 4. Correct. Basilar skull fractures are the most serious fracture. You see bleeding where? Eyes, ears, nose, and throat. So, you will see cerebrospinal rhinorrhea with a (+) Halo test. If you have a bloody spot on the sheet, or wherever, when CSF is present, it will settle out and form a ring or halo around the blood spot. Raccoon eyes, is perioribital bruising which is seen with a basilar skull fracture. Battle's sign or bruising over the mastoid is also indicative of a skull fracture. 2. Incorrect. Hyper-reflexia has to do with deep tendon reflexes.It is not a sign of a basal skull fracture. 5. Incorrect. Think meningitis with Kernig sign.

Which clinical manifestation does the nurse expect to see in a client diagnosed with Addison's disease? 1. Confusion 2. Hypertension 3. Vitiligo 4. Hyperkalemia 5. Hypernatremia 6. Weight gain

1., 3., & 4. Correct: Clients with Addison's disease may present with nonspecific symptoms of confusion. As the continual reduced functioning of the adrenal medulla and adrenal cortex occurs, the client will present with cognitive impairment, delusions, and hallucinations. The reduced blood cortisol increases the adrenocorticotropic hormones (ACTH) and the melanocyte-stimulating activity. The feedback mechanism results in the hyperpigmentation of skin. A deficiency of mineralocorticoids will result in the decreased excretion of potassium which results in hyperkalemia. 2. Incorrect: The client diagnosed with Addison's disease will present with hypotension. The decrease in the production of the adrenal cortex steroids results in the increased excretion of sodium. The sodium loss can cause severe dehydration, decreased circulation, and hypotension. 5. Incorrect: The increased excretion of sodium is the feedback action of the decreased level of adrenal cortex steroid. This action will result in hyponatremia. 6. Incorrect: The reduced glucocorticoid will result in weight loss not gain. This reduction is due to changes in the carbohydrate, fat, and protein metabolism.

A client is admitted with hypocalcemia. Which treatment would the nurse anticipate for this client? 1. PO Calcium 2. Rapid IV Push Calcium 3. Vitamin D 4. Sevelamer hydrochloride 5. Phosphate supplements

1., 3., & 4. Correct: Since this client has hypocalcemia, PO Calcium replacement would be an appropriate treatment. Now, let's look at the others that are not as obvious. Vitamin D helps to improve calcium absorption, which will help increase the calcium levels. So, what is sevelamer hydrochloride and how will this help hypocalcemia? Well, it is a phosphate binder. And remember that we said if you bind the phosphorus, the phosphorus levels go down. And since phosphorus and calcium have inverse relationships, as the phosphorus levels go down, the calcium levels will go up! 2. Incorrect: IV Calcium should be administered slowly or by slow infusion and the client should always be on a heart monitor. If you give calcium too rapidly by IV, the client may have vasodilation, hypotension bradycardia, cardiac arrhythmias, syncope, and cardiac arrest. Don't forget to be watching for the widening of the QRS complex when administering IV calcium! 5. Incorrect: Phosphate supplements would cause the calcium to be even lower in this client. Remember, phosphorus and calcium have an inverse relationship. We would give phosphate binders, not supplements.

The nurse is preparing discharge teaching instructions for a client post right radical mastectomy with reconstruction. What instruction should the nurse include? 1. Squeeze tennis ball with right hand every 2-4 hours while awake. 2. No blood pressure readings in right arm for one year. 3. Wear gloves when gardening. 4. Wear your watch on the left wrist. 5. Brush your hair with your left hand until pain free.

1., 3., & 4. Correct: Squeezing a tennis ball will help promote new circulation. Protect the hand and arm at all times. A tiny cut could turn into a major infection, so wearing gloves while gardening is a good idea. Since the mastectomy was on the right breast, the client can wear a watch on the left wrist. Do not wear anything constricting on the right wrist, or arm. 2. Incorrect: No blood pressure reading in right arm ever. 5. Incorrect: We want the client to use the affected arm when brushing hair. This will help promote new circulation and will help prevent frozen shoulder. So, this client should use the right hand to brush her hair.

A client's skeletal traction has been accidently released. What signs/symptoms does the nurse expect to see? 1. Pain 2. Foot drop 3. Muscle spasm 4. Bone displacement 5. Itching under the straps

1., 3., & 4. Correct: The purpose of traction is to stabilize and realign bone fractures and reduce pain. If the skeletal traction is interrupted by losing the traction on the bone, the result may include pain, muscle spasm, and bone displacement. 2. Incorrect: Foot drop is the weakness or paralysis of the muscles that lift the front part of the foot. Causes of foot drop may include; nerve injury, muscle or nerve disorders, brain and spinal cord disorders, and immobility. 5. Incorrect: The client would not experience any itching under any straps or cords due to the accidental release of the skeletal traction.

The nurse is developing the plan of care for a client admitted for the treatment of mania. Which interventions should the nurse include? 1. Give one cigarette to client at a time. 2. Discuss delusional belief with client. 3. Have finger foods available at mealtime. 4. Give high calorie fluids between meals. 5. Provide soothing music in room during waking hours.

1., 3., & 4. Correct: We need to protect this client from hazards in their environment. They have no control or awareness of these hazards. If they smoke, only give the client one or two cigarettes at a time, or the client will light a whole pack at once. Finger foods should be provided because the cleint is too busy to stop and eat. They are also too busy to drink, so they can become dehydrated. This is why we provide high calorie fluids for them throughout the day. 2. Incorrect:You are not supposed to talk a lot about the client's delusions. Let the client know that you accept their need for the belief, but that you do not believe it. 5. Incorrect: We want to decrease the stimuli in this client's environment, so that means turning off the TV and radio. Any stimulating activity needs to be interrupted.

A client has a history of deep vein thrombosis (DVT) and pulmonary embolism (PE). What should be included in the teaching by the nurse as preventive measures for the development of a DVT and PE? 1. Drink plenty of fluids on a daily basis. 2. Stop and move around every 4 hours when taking a long trip. 3. Perform isometric and stretching exercises in the lower extremities. 4. Need for weight management. 5. Walk around 4-6 times per day.

1., 3., 4., & 5. Correct: In order to get this question correct, you must first consider some of the risk factors for developing a DVT and PE. Some causes include: dehydration, venous stasis from prolonged immobility or surgery, obesity, birth control pills, clotting disorders, and heart arrhythmias like A-Fib. Therefore preventive measures would include such things as hydration by increasing fluid intake, prevention of stasis by isometric and stretching exercises of the feet, knees, and hips every 2 to 4 hours, prevention of obesity, and walking around at least 4 to 6 times per day. 2. The client does need to stop when traveling long distances, but it needs to be done more often than every 4 hours. This client has a history of DVT and PE, so it is very important that the client stop and move around at least every 2 hours.

A client has returned to the room post stem cell transplant. What early signs of rejection should the nurse monitor for in the client? 1. Abdominal pain 2. Straw colored urine 3. Jaundice 4. Pruritus 5. Diarrhea

1., 3., 4., & 5. These are early signs of rejection that the nurse must monitor for: abdominal pain, jaundice, pruritus or itching, and diarrhea. 2. Incorrect: There is nothing wrong with the color of this urine. A problem would be dark, tea colored urine. This indicates the breakdown of red blood cells.

What signs/symptoms does the nurse expect to see in a client who has ulcerative colitis? 1. Abdominal cramping 2. Hematemesis 3. Diarrhea 4. Fever 5. Rebound tenderness 6. Rectal bleeding

1., 3., 4., 5., & 6. Correct: Ulcerative colitis is an ulcerative inflammatory bowel disease in the large intestines. Common s/s include abdominal cramping, diarrhea, fever, rebound tenderness, and rectal bleeding. 2. Incorrect: Hematemesis is seen with upper GI bleeding.

A client in her first trimester of pregnancy has been attending educational sessions on pregnancy. What statements by the client would indicate to the nurse that client teaching has been successful? 1. "Good food sources of iron includes spinach, raisins, and dark chocolate." 2. "I will eat at least 40 grams of protein a day." 3. "Taking folic acid will help prevent heart defects from occurring." 4. "Swimming is an acceptable exercise for me while I am pregnant." 5. "I can gain 2 pounds (0.9 kg) per week during my first trimester." 6. "I need to stay out of hot tubs while pregnant."

1., 4., & 6. Correct: Good sources of iron include liver, spinach, lentils, raisins, fortified cereals, dark chocolate, and dried fruits. Walking and swimming are the best exercises for a pregnant woman. Remember, no high impact. We also do not want them to get overheated, so do not let mom get into hot tubes or under heating blankets because this will increase body temperature and can cause birth defects. 2. Incorrect: Pregnant women should increase protein intake to 60 grams per day. Here's the deal, normal protein intake is about 40 to 45 grams per day. But when you are pregnant, you have a lot of tissue growth going on in your body, so you need more protein. 3. Incorrect: Folic acid helps to prevent what type of defect? Neural Tube Defects. Spinal bifida or myelomeningocele and anencephaly are the big neutral tube defects. 5. Incorrect: During the entire 1st trimester, the client should gain no more than 4 pounds (1.8 kg).

A facility housekeeper approaches the nurse, reporting their sibling with no advanced directive has been admitted in a coma following a massive stroke. As the client's only family member, the housekeeper requests information on the client's condition and prognosis. What actions by the nurse are most appropriate? comfort. 2. Inform housekeeper that you are not the client's nurse. 3. Check the chart data and provide brief update on client. 4. Ask Social Services to help housekeeper with legal issues. 5. Offer to call primary healthcare provider for housekeeper.

1.,4., and 5. CORRECT. There are obvious legal issues which complicate this situation. Though the housekeeper claims to be the only living relative, unless this claim can be legally proven, the nurse cannot verify this information. However, the nurse can still assist the housekeeper or any family member with other needs. Contacting a spiritual leader of choice can be easily accomplished. Even more important is that Social Services, as part of an interdisciplinary team, can assist the housekeeper with multiple needs, including legal requirements to obtain guardianship or power of attorney. It would also be helpful to advise the primary healthcare provider that the housekeeper is the sole relative, and allow that physician to make a determination what to reveal under such dire circumstances. 2. INCORRECT. The nurse's statement may be correct but does not provide the housekeeper with usable information or alternatives to achieve the goal regarding sibling's condition. 3. INCORRECT. HIPAA prohibits accessing charts of client's unless directly involved with the care for that individual. Even if the nurse was assigned to the sibling, it is a violation to share information, even under the unique circumstances described in the scenario.

A client is admitted to the emergency department (ED) following blunt trauma to the chest from a motor vehicle accident. A hemothorax and pneumothorax are suspected. What signs and symptoms would the nurse anticipate recording to support this diagnosis? 1. Shortness of breath 2. Decreased heart rate 3. Wheezing in the affected area 4. Chest pain 5. Cough 6. Subcutaneous emphysema

1.,4.,5., & 6. Correct: With a hemothorax, we recall that blood has accumulated in the pleural space, and with a pneumothorax, we know that it is air that has accumulated in the pleural space. The presence of either of these causes the lung to collapse. The signs that the nurse expects to see includes shortness of breath, chest pain, and cough. We may also see subcutaneous emphysema as the air that escaped from the lung becomes trapped in the surrounding tissues. 2. Incorrect: What happens to gas exchange when there is a hemothorax or a pneumothorax? That's right! It decreases. When hypoxia is present, the body responds by increasing the heart rate. Therefore, we would not expect the heart rate to be decreased in this situation. 3. Incorrect: Wheezing occurs when air tries to get through narrowed passages. With a hemothorax or pneumothorax, the lung sounds will be diminished over the affected area where the lung has collapsed.

A new nurse is assigned to address quality improvement on a medical-surgical unit. The nurse is aware what tasks could not be safely completed by a UAP? 1. Obtain vitals on a client following a colonoscopy. 2. Reinforce teaching for a client awaiting discharge. 3. Feed pureed food to client with left-sided paralysis. 4. Get finger stick on confused client with diaphoresis. 5. Provide ice packs to client with a new long leg cast.

2, 3, and 4. CORRECT. Unlicensed assistive personnel, often referred to as a 'nursing assistant', should be assigned repetitive, uncomplicated tasks on stable clients. These tasks include activities of daily living, routine vital signs and ambulation. The UAP is not able to reinforce teaching because this requires an evaluation of client learning. The client with left-sided paralysis is at risk for aspiration and requires specific knowledge about position during feeding. An LPN or RN should be assigned to feed this client. Getting a blood sugar by finger-stick is not within the abilities of a UAP across the country. Some states allow FSBS by UAPs that have been trained, but this is not from coast to coast. 1. INCORRECT. Obtaining vitals on a post-procedure client is based on the type of test and potential for complications. A colonoscopy is generally an uncomplicated procedure and the UAP would certainly be able to do vitals on this client. 5. INCORRECT. Filling and providing ice packs to a client with a cast is within the scope of activities for unlicensed assistive personnel. The nurse would need to provide instruction about placing ice packs to the side of the cast, rather than on top, but this activity can definitely be assigned to the UAP.

The nurse performs a rapid assessment on a client who states, "I feel really sick and my heart is beating so fast." What signs and symptoms would indicate to the nurse that the client's cardiac output is inadequate? 1. CVP 5 mm Hg. 2. Moist skin. 3. Urinary output 150 mL over 4 hours. 4. Weak radial pulses. 5. BP 90/50, HR 200, RR 22. 6. Mild chest discomfort.

2, 4, 5, & 6. Correct: When cardiac output is inadequate, the vital organs are not being perfused properly. The skin will be cool and clammy (moist) because the skin is not being perfused. Radial pulses will be weak and thready, because less blood is pumping through the arteries. Less volume means less pressure, so BP is low. The heart rate is too fast, so blood does not have time to get into the ventricles before it is contracting again, which decreases cardiac output. Less blood is being pumped into the body. Chest pain means oxygenated blood is not reaching the heart muscles. 1. Incorrect: Normal CVP is 2-6 mmHg, so this is a normal finding. 3. Incorrect: Normal urinary output (UOP) should be at least 30 mL per hour. This client's UOP was 150 mL over 4 hours (37.5 mL per hour). So there is no concern here.

A client reports dizziness and weakness while walking down the hall. The nurse notes the client's cardiac rhythm displayed on the telemetry monitor. What actions should the nurse take? 1. Assist client in ambulating back to bed. 2. Obtain client's blood pressure. 3. Auscultate lung sounds. 4. Prepare for cardioversion. 5. Initiate 100% oxygen per nonrebreather mask.

2. & 3. Correct: The client is dizzy and weak. This client is at risk for falling, so think safety and get the client back in bed. Use a wheelchair to accomplish this. Then obtain the client's BP. It may be low, indicating poor tissue perfusion to the vital organs. One cause of premature ventricular contractions (PVCs) includes heart failure, so assess the lungs for adventitious sounds. 1. Incorrect: This client is dizzy and weak. Having the client ambulate back to the bed is a safety risk. The client could fall or the condition could deteriorate while ambulating. 4. Incorrect: Cardioversion is not indicated with an underlying rhythm that is normal (NSR) with PVCs. Oxygen may decrease the PVCs. If not, medication can be administered to decrease the rate of the PVCs. 5. Incorrect: Oxygen may abate the PVCs; however, it should be initiated at 2 liters/NC rather than at 100% per nonrebreather mask. Start with the least amount of oxygen that could relieve symptoms.

A client was diagnosed with a fractured ulna 8 hours ago. Which assessment data may indicate a compartment syndrome? 1. The pain is located at the elbow area. 2. The prescribed opioid does not relieve the pain. 3. When forearm is elevated, the swelling in the forearm is reduced. 4. The pain in the forearm is described as a 9 on a 10 scale and throbbing. 5. When placing a cold compress on the forearm, the pain level is reduced.

2. & 4. Correct: Compartment syndrome occurs when swelling occurs within the compartment. This results in increased pressure on the capillaries, nerves, and muscles in the compartment. The pain is very intense. The client is expressing pain at a 9 on a 10 scale and throbbing. The pain is also unrelieved by opioid administration. 1. Incorrect: The location of pain at the elbow area does not indicate the presence of compartment syndrome. The pain related to compartment syndrome would not occur in the elbow. The swelling and bleeding will occur in the compartment of the forearm due to the swelling or bleeding. 3. Incorrect: The swelling will not be reduced by elevating the forearm as result of the constant increased pressure in the compartment. 5. Incorrect: Applying a cold compress on the forearm that decreases the swelling is not a symptom of compartment syndrome. The increased pressure in the compartment results in a decrease of the blood flow to the muscles and nerves.

The nurse is reviewing the primary healthcare provider's (PHP) initial prescriptions for a client diagnosed with diabetic ketoacidosis (DKA)? Which prescription from the PHP would the nurse question? 1. Arterial blood gases 2. 500 ml D5W at 100 mL per hour 3. Serum glucose levels every hour 4. Hourly adjustment of Regular insulin IV according to serum glucose level protocol 5. 100 mL O.45% sodium chloride (NaCL) with potassium chloride KCL 10mEq IV

2. & 5. Correct: The clinical manifestation of DKA is a serum glucose level of greater than 300mg/dL. The goal of the treatment for DKA is to reduce the serum glucose level. Prescribing D5W will increase the client's serum glucose level which is already elevated. The prescription should begin with 0.9% NaCL. to compensate for the effects of polyuria, IV normal saline, an isotonic solution. An isotonic solution is composed of equal concentrations of solutes and water which will increase vascular volume. Initially the potassium is normal or high and can decrease when treatment begins. This prescription should be questioned. 1. Incorrect: The prescription for arterial blood gases is appropriate. The arterial blood gases will identify if the client is in metabolic acidosis. 3. Incorrect: The goal of the treatment for DKA is to reduce the elevated glucose level. The glucose levels of the client are evaluated hourly to monitor the efficiency of the treatment and assess for hypoglycemia. As the serum glucose level reduces to 250 -300 mg/dL the IV prescription will change from normal saline to D5W. 4. Incorrect: The glucose levels of the client are evaluated hourly to monitor the desired outcome of control of the serum glucose level. The Regular insulin prescription is adjusted according to the serum glucose levels. The objective of the Regular insulin prescription is to reduce the hyperglycemic episode without a hypoglycemic episode.

The charge nurse is assigning several immediate tasks to on-coming shift personnel. What task should the nurse assign to a licensed nurse only? 1. Reposition a client with a long term Peg tube. 2. Obtain scheduled vital signs during blood transfusion. 3. Assist diabetic client with neuropathy to walk to bathroom. 4. Ambulate client following laparoscopic appendectomy.

2. CORRECT. Blood transfusions present a potential for complications the entire time the blood is transfusing, even though any problems which might occur generally happen in the first 15 minutes. But data collection regarding client response, including skin color, respiratory status and IV site should be assessed during vitals sign monitoring. 1. INCORRECT. Repositioning a client with a Peg tube can be safely accomplished by unlicensed assistive personnel. A Peg tube is secured inside the stomach wall and simply repositioning a client would not create an issue requiring a nurse. 3. INCORRECT. Ambulating a diabetic client requires the same precautions as walking with any individual. Neuropathy may cause pain or numbness to the soles of the client's feet, but the UAP is still capable of ambulating this client safely. A licensed nurse is not required at this time. 4. INCORRECT. A laparoscopy is the process of performing a surgical procedure without cutting open the body. Three or four small holes are created through which instruments are passed, including a camera, allowing the surgery to be completed. Ambulating this client is no different from any other post-operative walk, and therefore can easily be accomplished by the UAP.

When working in a new facility, the nurse identifies several violations of client privacy and confidentiality. What situation should the nurse report immediately to the supervisor? 1. Primary healthcare provider left client chart opened on the desk while completing rounds. 2. Students in teaching hospital observe client care without permission. 3. Staff personnel in an elevator laughing about "crazy guy" on 4th floor. 4. Secretary copying client charts leaves several pages in office copier.

2. CORRECT. Client confidentiality and privacy are guaranteed by both federal and state regulations as well as those principles outlined in the nurse's code of ethics. As a client advocate, the nurse has the responsibility to protect those rights by reporting infractions to the appropriate personnel. An actual violation has occurred when students enter a client's room to observe a procedure without first asking the client's permission. Even though the client is in a teaching hospital, this does not eliminate the client's right to privacy or to refuse to have students present. 1. INCORRECT. Opened and unattended charts represent the potential for confidential information to be viewed by others, including non-medical persons. The potential exists for client information to be seen or used by those not entitled to view such information. Charts should remain closed when not in use. 3. INCORRECT. Such a breach of ethics violates multiple healthcare principles. However, though unprofessional and rude, the staff did not mention a client name or room number. That does not mean there will be no disciplinary action, but actual client details were not revealed. 4. INCORRECT. Leaving client data in a copier violates client privacy and a potential breach of confidentiality. The copy machine is in facility office, not a public location; however, there are individuals within an office which should not have access to those client records.

A client had radiation seeds implanted to treat prostate cancer. When entering the room to initiate discharge teaching, the nurse observes the spouse emptying the client's urinal. What is the nurse's priority action? 1. Immediately escort spouse to ED to check radiation levels. 2. Begin discharge teaching to the client and spouse. 3. Have spouse wash hands thoroughly and apply sterile gloves. 4. Explain that spouse must remain outside the room until urinal is emptied.

2. CORRECT. Internal radiation, also called brachytherapy, is placed inside the body as close to the cancer as possible. Internal radiation therapy can be permanent or temporary as well as sealed or unsealed, which refers to the amount of radiation risk posed by the client. Implanted seeds used to treat prostate cancer are a type of sealed radiation, indicating the body fluids are not radioactive. Emptying the urinal poses no risk to the spouse. 1. INCORRECT. Implanted seeds are a type of sealed radiation. Therefore, the client's body fluids are not radioactive, though the spouse should use some precautions when in proximity to the client for a few days. No need to check the spouse for radiation levels. 3. INCORRECT. Even though the client will be immunosuppressed, there is no need for the spouse to use sterile gloves. However, washing hands and using regular gloves is always a good idea. 4. INCORRECT. The spouse does not need to remain outside the room, particularly since the client is about to be discharged home. The client's body fluids are not radioactive.

In what position should the nurse place a client post lumbar puncture? 1. Reverse trendelenburg 2. Prone 3. Side-lying 4. Supine HOB elevated 45 degrees

2. Correct. Post lumbar puncture, the client should lie flat or preferably prone for 4-8 hours so that a seal will form at the puncture site. 1. Incorrect. What possition is reverse trendelenburg? Trendelenburg is head down and legs up. Reverse trendelenburg is HOB up and feet down. Not the position we want this client to be placed. 3. Incorrect. Not side-lying. Flat or prone so that a seal will form. 4. Incorrect. Do not elevate the HOB for 4-8 hours.

A client, with a T5 injury, has not had a bowel movement in three days. Today, the client reports a headache rated 10/10. The nurse takes the client's vital signs: BP 180/110, HR 52, RR 20. What action by the nurse takes priority? 1. Administer hydralazine 20 mg IV. 2. Elevate head of bed 45 degrees. 3. Remove impaction with topical anesthetic. 4. Close air vents in the room.

2. Correct. These signs/symptoms should lead the nurse to realize that the client is experiencing autonomic dysreflexia. The priority is to lower the blood pressure by raising the head of the bed to a semi-fowler's position. 1. Incorrect. You may have to give antihypertensive medications, but first elevate the head of the bed. 3. Incorrect. You have to remove the stimuli, but first get that BP down, so the client does not have a hypertensive stroke. 4. Incorrect. Again, drafts can cause autonomic dysreflexia, but the priority is to decrease that BP.

A client who has been given steroids for a prolonged period to treat asthma, reports dizziness, tingling of the fingers, and muscle weakness. What action should the nurse take first? 1. Determine current blood pressure 2. Connect client to a cardiac monitor 3. Administer oxygen 4. Obtain arterial blood gases

2. Correct. These symptoms are indicative of hypokalemia and metabolic alkalosis. What do steroids do to the body? Steroids make you retain sodium and excrete potassium. So, you could become hypokalemic. Low potassium levels cause an increase in the reabsorption of bicarb by the kidneys. That is why you sometimes see metabolic alkalosis with Cushing's disease and prolonged steroid use. What electrolyte imbalance do we see with metabolic alkalosis? It's hypokalemia. So, if you have a client who is hypokalemic then they may have muscle weakness, hypotension and life threatening arrhythmias. And we know when the potassium is messed up, we should always think about the heart first. Connect the client to the cardiac monitor. 1. Incorrect. The priority is going to be checking the heart rhythm because a low potassium can cause a life-threatening arrhythmia. 3. Incorrect. The symptoms are most likely due to low potassium levels. This could lead to life-threatening arrhythmias. How would you fix this problem? Yes, give potassium, not oxygen. 4. Incorrect. You can do this after you check the heart rhythm. The priority is going to be checking the heart rhythm because a low potassium can cause a life-threatening arrhythmia.

The nurse is providing dietary instructions to a client newly diagnosed with type 2 diabetes. Which food examples should make up the highest percentage of this client's recommended diet? 1. Pecans, eggs, pork chop 2. Asparagus, broccoli, cabbage, and cucumbers. 3. Lean hamburger, fish, skinless chicken 4. Whole milk, cheese, dark chocolate

2. Correct: A calorie is the unit of energy needed to raise the temperature of 1 kilogram of water 1 degree of Celsius. The recommended percentage of calories from carbohydrates is 50% of the daily diet. Nonstarchy vegetables are lower in carbohydrates, so they do not raise blood sugar very much. They are also high in vitamins, minerals, and fiber, making them an important part of a healthy diet. Filling half your plate with nonstarchy vegetables means you will get plenty of servings of these superfoods. Examples include asparagus, broccoli, cabbage, brussel sprouts, carrots, and cucumbers. 1. Incorrect: The listed foods are high in protein. The intake of proteins will reduce appetite because protein takes longer to digest in the stomach. This results in a person feeling fuller for a extended period. The recommended percentage of daily calories from proteins is 20-25% for both a regular diet and a diabetic diet. Proteins are beneficial to build, repair, and maintain the body's tissues. Protein with high levels of fat can cause both a weight gain and an increase in the serum glucose levels. Also a high intake of protein has not proven to influence the level of serum glucose levels. 3. Incorrect: The foods that are listed are high in protein, which should consist of 20-25% of the diabetic diet. The intake of various amounts of high protein foods will not affect the client's glucose level. 4.Incorrect: The monounsaturated and polyunsaturated fats such as milk, cheese and chocolate in the diet function such as components of cell membranes, energy storage, energy storage, and fat-soluble vitamin A, D, E and K. The intake of fat does not cause an increase or decrease in serum glucose levels.

The charge nurse in a psychiatric facility is assigning morning tasks to an unlicensed assistive personnel (UAP). What task should the nurse instruct the UAP to complete first? 1. Accompany client off unit to smoking area. 2. Obtain a morning weight on anorexic client. 3. Assist a client who is depressed to get out of bed. 4. Prepare the day room for group breakfast.

2. Correct: An accurate daily weight is obtained each morning at the same time, on the same scale, in the same clothing. The accuracy of this procedure is particularly critical for the anorexic client and should be performed prior to breakfast. The nurse will also remind the UAP to be particularly vigilant of the client attempting to alter the scale reading, perhaps by hiding an object in a bathrobe pocket. 1. Incorrect: Clients who smoke often request an early morning cigarette, prior to breakfast, and must be accompanied by a member of the staff during that time. Although many facilities are non-smoking, older clients who do smoke are provided with a specific location to do so, but this is not a priority at this time. 3. Incorrect: It is important to help clients who are depressed to participate in daily routines, such as eating breakfast in a group setting. Based on the degree of depression, many clients may also require assistance to even get out of bed and dress. The UAP will need to complete this task before breakfast but it is not the first priority of the morning. 4. Incorrect: It will be important to get the day room ready for group breakfast. However, preparing the dayroom will likely take quite a bit of time and there is a more important task that needs to be completed prior to breakfast preparations.

The nurse is caring for a client being admitted to the emergency department after being stabbed in the chest. An occlusive dressing is covering the chest wound upon arrival. The client's condition begins to deteriorate. Assessment reveals tracheal deviation, diminished breath sounds bilaterally, and asymmetrical chest wall movement. What would be the priority nursing intervention? 1. Administer high flow O2 per face mask. 2. Remove the occlusive dressing. 3. Notify the healthcare provider. 4. Initiate rapid IV resuscitation.

2. Correct: Based on these signs and symptoms, we recognize that the client has developed a tension pneumothorax due to the occlusive dressing not only preventing air from getting in, but not allowing the air to escape. Therefore, management of this emergency situation would be the nurse's priority. Removal of the occlusive dressing would allow the air to escape and should help reduce the pressure that is causing the mediastinal shift. Needle decompression may also be needed to relieve the tension pneumothorax. If available, a good option for covering an open or "sucking" chest wound would be a petroleum gauze dressing which would be taped down on only three sides. This creates a flutter valve mechanism that allows air to escape but prevents air from re-entering through the open wound. 1. Incorrect: Although oxygen is used in clients experiencing hypoxia, it would not help to relieve or fix the tension pneumothorax. 3. Incorrect: This is an emergency and you, as the nurse, should recognize that the occlusive dressing has been trapping the air inside the chest and has created an emergency. You should carry out the intervention that you know will help to relieve the pressure that is building up as a result of air trapping in order to prevent further deterioration of the client's condition. Therefore, calling the healthcare provider first would be a delay of emergency care. 4. Incorrect: Did you see the word "deteriorating" and think shock and fluid resuscitation? The problem here is the trapping of air that is causing the tension pneumothorax. Administering IV fluids will not help to fix the problem.

A nurse has performed teaching with a client diagnosed with Cushing's disease. Which statement by the client would best indicate understanding of the teaching? 1. "The increased level of ADH will cause my potassium level to be too high." 2. "I will be retaining sodium and water due to the increased amount of aldosterone." 3. "I will be losing lots of fluid due to the hormonal imbalance I have." 4. "I will feel jittery and nervous due to the elevated thyroxine levels."

2. Correct: Cushing's is a disease that results in increased secretion of aldosterone. Having too much aldosterone causes the client to be at risk for fluid volume excess (FVE) due to the increased retention of both sodium and water. 1. Incorrect: Cushing's is a problem associated with an increased production of aldosterone, not ADH. The client will be retaining both sodium and water. 3. Incorrect: The client would not be losing excess fluid as is seen in clients with Diabetes Insipidus (DI), an ADH problem. The client will be retaining both sodium and water due to the increased aldosterone and would be at risk for fluid volume excess. 4. Incorrect: Increased thyroxine levels is related to hyperthyroidism, not Cushing's disease. This client has a problem with too much aldosterone and a resulting FVE.

A client is admitted with prolonged nausea and vomiting. The client's admission sodium level is 149 mEq/L (149 mmol/L). What action by the nurse would be most appropriate at this time? 1. Administer 3% NS at 150 mL/hr 2. Perform neurological assessment 3. Increase oral intake of sodium 4. Decrease fluid intake

2. Correct: Did you recognize that the sodium level of 149 is too high? The normal sodium level is 135-145 mEq/L (135-145 mmol/L). Think about the testing strategy that we mentioned to you. Look for neuro changes when the sodium level is not within normal limits. The brain does not like it when the sodium level is messed up. So, performing a neurological assessment on this client would be important. 1. Incorrect: What type of fluid is 3% NS? It's a hypertonic solution that contains a lot of sodium! That would be a killer answer here because this client's sodium level is already too high! 3. Incorrect: The sodium level is too high. The nurse would have the client to decrease, not increase, the oral intake of sodium. 4. Incorrect. With hypernatremia, there is too much sodium and not enough fluid. Therefore, you would want this client to increase, not decrease, the fluid intake to dilute the sodium level in the blood.

What is the best position for the nurse to place a client for a thoracentesis of the right lung? 1. Lying supine with pillow removed and head of bed flat 2. Sitting on side of bed and leaning over the bedside table 3. Lying on the right side with the head of bed at 45 degrees 4. Lying supine with the left arm raised over the head

2. Correct: For maximum accessibility for the thoracentesis to be performed, the client should be positioned in a sitting position on the side of the bed, leaning over a bedside table, with arms propped on pillows and the feet supported. If the client is not able to sit up, the alternative position would be to lie on the unaffected side with the head of bed elevated 45 degrees. 1. Incorrect: The approach for a thoracentesis is generally a posterior approach, so the supine position would not be optimal. Also, if a client has a respiratory problem requiring a thoracentesis, the client may experience increased respiratory distress if placed in a supine position. You would never want to place a client in a position that would increase the respiratory effort or cause distress. 3. Incorrect: If the client is unable to sit up on the side of the bed, it is acceptable to be in a side lying position with the head of bed elevated 45 degrees. However, the client should be placed on the unaffected side, which in this case would be the left side. In this client, the right side is the affected side and should be in the superior position. 4. Incorrect: The best position is sitting up, leaning over the bedside table, with arms resting on pillows and the feet supported. The alternative position is side lying, with the head of bed elevated. The supine position is not a position of choice but may have to be used under certain circumstances. If placed in a supine position, the arm on the affected side would be placed over the head. In this case, the right arm would need to be raised, not the left arm.

The nurse is preparing to perform Leopold maneuvers on a newly admitted laboring client. What should the nurse remember when performing this procedure? 1. Ask the client to drink water prior to the procedure. 2. Perform procedure between contractions. 3. Monitor for heart beat acceleration with fetal movement. 4. Connect client to fetal heart monitor.

2. Correct: If you try to do the maneuvers during a contraction, what are you going to feel? A hard uterus. You're not going to feel the baby. 1. Incorrect: You want to have the client drink water to distend the bladder prior to an ultrasound of the uterus to bring it closer to the surface. For Leopold maneuvers we want the client to void first. A distended bladder can push the uterus to one side. 3. Incorrect: With a non-stress test, you look for two or more acceleration of 15 beats per minute or more with fetal movement. 4. Incorrect: This test does not require the client to be connected to the fetal monitor. What you're doing with Leopold maneuvers is you're palpating around the abdomen trying to find the baby's head, back, booty. Why do I care where the baby's back is? That's where you listen for the fetal heart rate.

The nurse completed discharge teaching on a client with two fractured ribs. Which statement by the client would indicate the need for further teaching? 1. "I will take deep breaths using my incentive spirometer every 2 hours." 2. "I will wrap my chest in an elastic bandage to support and immobilize my ribs." 3. "I will talk to my healthcare provider before taking the narcotic pain medicine that I currently have at home." 4. "I will notify my healthcare provider if I develop any change in my respirations or secretions."

2. Correct: Immobilizing, and therefore restricting the chest wall movement, with binders and straps is not recommended as it leads to shallow breathing, atelectasis, and pneumonia. The client should be taught to use the hands to support the injured area. 1. Incorrect: This would be an appropriate statement by the client and would be an indicator that the teaching was effective. Incentive spirometry is used to help prevent respiratory complications such as pneumonia and respiratory acidosis. 3. Incorrect: This would be an appropriate statement by the client and would be an indicator that the teaching was effective. Clients with rib fractures are generally prescribed non-narcotic analgesics. This is done to avoid narcotics suppressing the respirations even more than what occurs with the reluctance to deep breathe associated with the painful rib fractures. Don't you agree that the client needs to deep breathe? Some clients may have other conditions being treated with narcotic pain medications. Before continuing these at home, the client should discuss this with the healthcare provider. If the pain is severe, a nerve block by anesthesia may be needed to facilitate deep breathing and coughing. 4. Incorrect: This would be an appropriate statement by the client and would be an indicator that the teaching was effective. Do you recall some of the possible complications associated with rib fractures? They include pneumonia, respiratory acidosis, pneumothorax, and hemothorax. Therefore, the client should notify the healthcare provider if respiratory difficulty develops, secretions increase or change color, cough develops or worsens, or other respiratory symptoms develop.

A client is admitted to a chemical dependency unit for addiction treatment. Which of the client's belongings should the nurse remove from the client's room? 1. Shampoo and conditioner 2. Mouthwash and hand sanitizer 3. Toothpaste and dental floss 4. Lotion and foot powder

2. Correct: Mouthwash and hand sanitizers have alcohol in them, which the client may drink. 1. Incorrect: There is not alcohol content in shampoo and conditioner, so these items do not have to be removed from the client's room. 3. Incorrect: We do not want the client to come into contact with anything that contains alcohol. Toothpaste and dental floss are safe for the client to have available. 4. Incorrect: There is no alcohol content in these items, so there is no need to remove them from the room.

The nurse performs an assessment on a client who reports abdominal pain. Based on the assessment findings, what problem does the nurse suspect? Awake, alert, and oriented reporting diffuse abdominal pain rated 9/10. Skin warm and dry. Cullen's sign noted. Abdomen rigid with guarding. Temperature 101 degrees F (38.3 degrees C), BP 96/64, HR 102, RR 26. 1. Cirrhosis 2. Pancreatitis 3. Peptic ulcer 4. Ulcerative colitis

2. Correct: These s/s point to pancreatitis. Look at the big clues: Cullen's sign, rigid abdomen with guarding, and fever. 1. Incorrect: What are the classic s/s of cirrhosis that are different from pancreatitis? Firm, nodular liver, dyspepsia, change in bowel habits, splenomegaly, acites. 3. Incorrect: Peptic ulcers typically do not present with severe pain, but with a burning pain in the mid-epigastric area and back. Dyspepsia is common as well, but no bruising around the flank area or umbilicus. 4. Incorrect: Ulcerative colitis presents with diarrhea, rectal bleeding, vomiting, weight loss, cramping, rebound tenderness and fever.

A client arrives at the clinic reporting a sharp pain, rated 10/10, radiating from the right flank around to the lower right abdomen. The client also reports nausea and vomiting. Based on this data, what problem does the nurse suspect? 1. Glomerulonephritis 2. Renal lithiasis 3. Nephrotic syndrome 4. Acute kidney injury

2. Correct: These signs and symptoms are classic for renal lithiasis or kidney stones. 1. Incorrect: Glomerulonephritis does present with flank pain but not as severe as a kidney stone and not radiating to the lower abdomen. There is no mention of edema seen with this client. 3. Incorrect: What do you see with neprhotic syndrome? Massive edema or anasarca. 4. Incorrect: Nausea and vomiting can occur with acute kidney injury because of all of the retained toxins. But there is no mention of hypertension, edema, and other s/s of acute kidney injury.

A parent voices concern because the 6 year child has not been eating as much in the last 3 months. What response from the nurse would be appropriate? 1. "You need to make the child eat more frequent meals to avoid becoming anorexic." 2. "This is not unusual in this age child because the growth rate has slowed down." 3. "Try providing high calorie foods that the child likes to increase the calories to 3500 per day." 4. "You are just being overly cautious. There is no need to worry about how much the child eats."

2. Correct: Think about normal growth and development here. Remember that the growth rate slows down in the school age child between 6 and 12 years of age. Therefore, they may not seem as hungry as they are during periods of growth spurts. 1. Incorrect: Forcing a child to eat can cause aversions to foods. There is nothing in the stem to indicate that the child is losing an abnormal amount of weight or is showing signs of anorexia. Although it can start this early, anorexia is not a problem that is generally seen in this age child. 3. Incorrect: This 6 year old child needs foods that are healthy but yet provide the calories needed. However, since the growth rate has slowed, the caloric needs at this age is about 2400 calories per day. Giving high calorie foods and increasing this to 3500 calories per day could lead to unhealthy weight gain and poor overall nutrition. 4. Incorrect: This response dismisses the parent's concern and does not address the issue. The parent should be provided with an explanation that this age child's appetite often decreases in relation to a decrease in the growth rate. However, the parent should be told that if weight loss or other problems begin to be noted, further evaluation may be needed.

A client presents to the emergency department (ED) with flu symptoms, fever, and chills. The nurse notes that the vital signs are: T 102.8°F (39.3°C), P 128, RR 30, B/P 154/88. ABG results are: pH-7.5, PaCO2 32, HCO3 23. What acid/base imbalance does the nurse determine that this client has developed? 1. Respiratory acidosis 2. Respiratory alkalosis 3. Metabolic acidosis 4. Metabolic alkalosis

2. Correct: This client has a high fever. Hyperventilation due to anxiety, pain, shock, severe infection, fever, and liver failure can lead to respiratory alkalosis. Here, the ABGs reflect respiratory alkalosis. pH > 7.45, PCO2 < 35, HCO3 normal. 1. Incorrect: The client is hyperventilating so CO2 (acid) is being blown off. The pH says alkalosis.3. Incorrect: Not a metabolic problem since the HCO​3 is in normal range and remember the pH says alkalosis.4. Incorrect: Not a metabolic related acid/base imbalance since the HCO3 is in normal range.

A teen male was diagnosed with infectious mononucleosis. What would be of most concern for the nurse when performing a history on this client? 1. Rides a bicycle three times a week 2. Plays on the varsity football team 3. Member of the swim team 4. Dances with the performing arts group

2. Correct: With infectious mononucleosis, the liver and spleen are often enlarged. Therefore, participation in contact sports should be limited to prevent injury. We worry about splenic rupture with contact sports such as football. 1. Incorrect: Although many activities are reduced in the acute phase due to fatigue and general malaise, riding a bicycle would not be as potentially dangerous as contact sports. 3. Incorrect: Swimming is considered a low impact sport. The client may be on bedrest or have limited activity due to fatigue. The client may need to delay swimming activities during the acute phase, but this would not be as dangerous as participation in contact sports. 4. Incorrect: Dancing can be strenuous. The client may be on bedrest or have limited activity due to fatigue. The client may need to delay strenuous activities during the acute phase, but this would not be as dangerous as participation in contact sports.

Assessment of a trauma client in the emergency department reveals paradoxical chest wall movement, respiratory distress, cyanosis, and tachycardia. The family is asking why the client needs positive end-expiratory pressure (PEEP). What should the nurse inform them regarding the rationale for this treatment? 1. Ventilation is improved as positive pressure is exerted into the airways as the client begins to take in a breath. 2. Gas exchange is improved, and the work of breathing is decreased. 3. It expands and realigns the ribs to aid in the healing process. 4. Allows for positive pressure to be applied continuously during inspiration and expiration. 5. It is less invasive and does not require the client to be on the ventilator.

2., & 3. Correct: I hope that you were able to recognize that the signs and symptoms are characteristic of a flail chest. This occurs with multiple rib fractures. The client will have pain, be anxious, and short of breath. The classic sign of a flail chest is the paradoxical (see-saw) chest movement in which the affected part of the chest sucks inwardly on inspiration and puffs out on expiration (opposite of what the normal side is doing). Dyspnea, cyanosis, and tachycardia are also generally seen. So, what is done about this unstable chest? PEEP may be used because it helps to improve gas exchange and decreases the work of breathing. As it exerts pressure in the lungs, it also facilitates the expansion and realignment of the ribs so that they can start growing back together. 1. Incorrect: The ventilator exerts the positive pressure down into the lungs at the end of expiration to keep the alveoli open. 4. Incorrect: Do you see the word "continuously"? This describes continuous positive airway pressure (CPAP). This is often used for individuals with sleep apnea and infants with underdeveloped lungs. 5. Incorrect: With PEEP, the client is on the ventilator.

The nurse is planning care for a client admitted for chemotherapy. What interventions should the nurse initiate to prevent infection? 1. Change IV tubing every 48 hours. 2. Place supplies for client in room. 3. Limit nursing personnel in room. 4. Bathe perineum once daily. 5. Place in protective isolation.

2., & 3. Correct: They need their own cups; they need their own everything. You don't need to go to a general closet and get supplies for this client. They need their own blood pressure cuff and own stethoscope in the room. Their own stuff because you only want their bacteria in the room. Limit people in the room who could pass on an infection to the client. Limit visitors, nurses, and nursing personnel to only those necessary to care for the client. 1. Incorrect: IV tubings should be changed daily, not every 2 days. 4. Incorrect: Bathe warm moist areas like underarms, groin, and perineum twice a day. Moist areas are a great place for bacteria to grow. 5. Incorrect: There is no indication that this client needs to be in isolation. A private room is acceptable at this time.

The nurse is providing teaching to a group of clients newly diagnosed with chronic stable angina. What points should the nurse include? 1. Wait 1/2-1 hour after eating to exercise. 2. Attend classes such as guided imagery to reduce stress. 3. Temperature extremes can precipitate an angina attack. 4. Gradually increase weightlifting training to improve cardiac output. 5. Eat a low fat, low fiber diet to lose weight. 6. Medications prescribed to prevent angina work by increasing the workload of the heart.

2., & 3. Correct: We want to teach clients who have angina to do whatever they can to decrease the workload of the heart. Stress can increase the workload on the heart, so learning ways to decrease or deal with stress is a positive step. This can be done through guided imagery or music therapy. Temperature extremes can precipitate an attack, so the client should dress warmly in cold weather and be cautious in extremely hot weather. 1. Incorrect: The client should wait at least 2 hours after eating to exercise. During this time, more blood goes to the digestive system. We don't want the heart to have to compete with the gut. 4. Incorrect: Weightlifting will increase the workload of the heart. We don't want to increase the workload of the heart in a client with a cardiac issue. 5. Incorrect: Losing weight is often beneficial for the cardiac client, so we advise them to decrease calorie consumption and maintain a low fat, high fiber diet. 6. Incorrect: We want to decrease the workload of the heart, not increase it. Medications are prescribed to prevent angina work to decrease the workload of the heart.

A nurse is caring for a client with a possible diagnosis hyperparathyroidism. Which serum laboratory value would validate this diagnosis? 1. BUN 12 mg/dL (4.28 mmol/L) 2. Calcium 12 mg/dL (3 mmol/L) 3. Sodium 140 mg/dL (140 mmol/L) 4. Phosphate 2.8 mg/dL (0.9 mmol/L) 5. Potassium 3.5 mEq/L (3.5 mmol/L)

2., & 4 Correct: Normal calcium range is 9.0 -10.5 mg/dl (2.25-2.62 mmol/L). The client's calcium level is 12 mg/dL (3 mmol/L) which is above normal range. Parathyroids secrete parathormone (PTH) for remodeling of the bones. PTH stimulates transfer calcium from the bone to the blood. Parathyroidism, an excess of PTH production by the parathyroids, will result in an increase in calcium movement from the bone to the blood. The normal range for phosphate is 3.0 - 4.5 mg/dL (0.97-1.45 mmol/L). The client's phosphate level is 2.8 mg/dL (0.9 mmol/L) which is below normal range. Parathyroidism, an excess of PTH production by the parathyroids, reduces the reabsorption of phosphorus in the kidneys. The result is that there is an increase in the excretion of phosphorus in the urine resulting in a decreased serum phosphorus level. 1. Incorrect: The normal range for BUN is 10-20 mg/dl (3.6-7.1 mmol/L). The client's BUN level is 12 mg/dL (4.28 mmol/L). 3. Incorrect: The normal range for sodium is 135 - 145 mEq/L (135-145 mmol/L). The client's sodium level is 140 mEq/dL (140 mmol/L). 5. Incorrect: The normal range for potassium is 3.5 -5.0 mEq/L (3.5-5.0 mmol/L). The client's potassium level is 3.5 mEq/dL (3.5 mmol/L).

A client comes to the clinic and states that she believes she is pregnant. What probable signs of pregnancy does the nurse expect to see? 1. Amenorrhea 2. Facial chloasma 3. Fetal movement 4. Breast tenderness 5. Positive pregnancy test 6. Urinary frequency

2., & 5. Correct: Probable signs are things that most likely indicate pregnancy and are signs the primary healthcare provider will identify. Facial chloasma, also known as the mask of pregnancy, is a probable sign. A positive pregnancy test is also a probable sign of pregnancy. Why isn't it a positive sign of pregnancy? There are other conditions that can increase hCG levels. 1. Incorrect: Amenorrhea is a presumptive sign of pregnancy. What does the word presumptive sign mean? Well, these signs suggest or indicate pregnancy but they could be attributed to something else. Presumptive signs are things that the client will recognize. 3. Incorrect: Fetal movement felt by the nurse is a positive sign. 4. Incorrect: Breast tenderness is a presumptive sign. 6. Incorrect: Urinary frequency is a presumptive sign.

A client is to begin external beam radiation for Ewing's sarcoma. What symptoms would the nurse teach the client to expect during radiation treatments? 1. Nausea and Vomiting 2. Skin shedding 3. Erythema with pain 4. Pancytopenia 5. Exhaustion

2., 3, 4 and 5. CORRECT. External beam radiation uses high energy proton rays to deliver radiation from outside the body. This therapy prevents cell reproduction and destroys cancer cells. Expected side effects can be topical or physiological, depending on the area radiated. Skin radiated by the beam becomes reddened (erythema), dry and peeling. Shedding skin and even blistering may occur because of multiple treatments. As radiation enters tissues, damage affects even healthy tissue like bone marrow. The client may eventually develop pancytopenia: a lack of all blood components, including red cells, white cells and platelets. As the body struggles with cancer and the effects of radiation, the client may experience severe or overwhelming fatigue which needs reported to the primary healthcare provider. 1. INCORRECT. Nausea and vomiting, along with other gastrointestinal symptoms, are usually associated with the use of chemotherapy and not necessarily radiation therapy.

The nurse is educating a client diagnosed with cirrhosis about the functions of the liver. What functions should the nurse include? 1. Removes old RBCs from the body. 2. Produces clotting factors. 3. Detoxifies the body. 4. Releases digestive enzymes. 5. Breaks down medications.

2., 3., & 5. Correct: Three of the four functions are listed: the liver produces clotting factors, detoxifies the body, and breaks down medications. It also synthesizes albumin. 1. Incorrect: The spleen, not the liver, removes old RBCs from the body. 4. Incorrect: The exocrine function of the pancreas releases digestive enzymes into the small intestine.

A primary healthcare provider has prescribed restraints for a violent adult client. Which measures would the nurse provide as proper interventions for this client? 1. Observe the client in restraints every hour. 2. Ensure that circulation to extremities is not compromised. 3. Assist client with needs related to nutrition and elimination. 4. Provide help with personal hygiene. 5. Renew restraint prescription in 4 hours if needed.

2., 3., 4. & 5. Correct: These are correct interventions for safety when a violent client requires restraints. When applying restraints you do not want the restraint so tight that extremity circulation is diminished. The client must still be provided with proper nutrition, hydration, and allowed to go to the restroom. If the client is restrained, the client will need help with basic care and comfort measures. Prescriptions for restraints used on an adult client must be renewed every 4 hours if needed. 1. Incorrect: The client in restraints should be observed every 15 minutes. Safety of the client is extremely important. Physical needs, such as food and toileting, should also be addressed.

A client receiving chemotherapy reports nausea and vomiting after every treatment. What interventions should the nurse initiate to reduce this side effect? 1. Administer antiemetic immediately after treatment. 2. Provide music therapy. 3. Provide ginger ale to drink. 4. Apply accupressure bands to wrists. 5. Place pepperment essential oil diffuser in room.

2., 3., 4., & 5. Correct: All of these interventions will help prevent or decrease nausea and vomiting. Music therapy is a form of behavioral therapy that can help with relaxation and distraction. Ginger is a natural antiemetic, so providing ginger ale to drink is beneficial. Sea bands or accupressure bands on the wrist help to relieve nausea and vomiting. Peppermint is one essential oil that relieves nausea. It can be used in a diffuser so that client can small the peppermint. 1. Incorrect: Antiemetic medications should be given one hour prior to chemotherapy rather than after receiving chemotherapy.

The nurse is initiating a client assessment. What signs and symptoms would validate the client's diagnosis of Cushing's disease? 1. Hypoglycemia 2. Mood alterations 3. Lipolysis 4. Truncal obesity 5. Hirsutism 6. Hyperkalemia

2., 3., 4., & 5. Correct: The client will experience mood swings. Several of the clinical manifestations of Cushing's are related to significant physical changes which can result in periods of depression for the client. Another clinical manifestation is lipolysis which is the breakdown of adipose tissue and the thinning of the extremities. Truncal obesity (apple-shaped obesity) is the distribution of adipose tissue located in the abdominal area. Hirsutism is when a female develops male characteristics such as increased hair on the face. When the adrenal cortex is stimulated there is an increase production of adrenal androgen. This results in the increased production of testosterone, a sex hormone. 1. Incorrect: The clinical manifestation of Cushing's is hyperglycemia not hypoglycemia. The increase adrenocortical activity in the adrenal cortex will result in hyperglycemia. 6. Incorrect: Client's diagnosed with Cushing's disease will present with hypokalemia not hyperkalemia. This is the result from increased adrenocortical activity which results in a decrease in potassium levels, hypokalemia.

A client is admitted following a severe burn. What changes related to fluid status would the nurse anticipate? 1. Fluid volume excess 2. Hypovolemia 3. Third spacing 4. Increased urine output 5. Low CVP 6. Increased urine specific gravity

2., 3., 5., & 6 Correct: Causes of fluid volume deficit (hypovolemia) include loss of fluid from anywhere as well as third spacing of fluid that occurs with such things as burns. Burns can result in fluid loss from the burn area as well as the third spacing, which increases the risk for hypovolemia and shock. As the fluid volume decreases, the BP and CVP both decrease. Remember, less volume, less pressure. Also, when the fluid volume becomes depleted, the urine output will decrease in an effort to hold on to the fluid (compensate) or the kidneys are not being perfused. You will see the urine specific gravity increase because the small amount of urine being produced will be very concentrated. 1. Incorrect: The client with a severe burn will lose fluids from the burn area and will also third space fluid to a place that does them no good. Therefore, they will go into a fluid volume deficit, not a fluid volume excess. 4. Incorrect: When the fluid volume becomes depleted, such as what occurs with burns, the urine output will decrease in an effort to hold on to the fluid (compensate) or the kidneys are not being perfused.

A nurse is providing teaching to parents of a child diagnosed with Cystic Fibrosis. Which teaching points should the nurse include? 1. Liver enzyme replacements must be administered to aid in digestion. 2. A well-balanced, high fat, and high calorie diet is important. 3. Water-miscible forms of fat soluble vitamins A, D, E, and K will be needed. 4. Pancreatic enzymes should be taken at least 1 hour after meals. 5. Respiratory and GI systems are often affected by thick, sticky secretions. 6. Both parents have the gene for this autosomal recessive disorder.

2., 3., 5., & 6 Correct: Nutrition is a major part of the care for clients with Cystic Fibrosis (CF). These clients are often underweight due to digestive problems. Fats are easier to digest than proteins or carbohydrates and provide more calories than other foods do. Increased pancreatic enzyme replacements will be needed as more foods that are higher in fat are consumed. Since these clients do not absorb fat well, the water-miscible forms of the fat soluble vitamins are needed. CF affects the exocrine glands. The mucous secretions are thick and sticky which often leads to problems and blockage in the respiratory and GI systems. Because it is a genetic disease with autosomal recessive transmission, this means that both parents must have the gene. The parents would need to know this when considering future pregnancies. 1. Incorrect: Pancreatic enzymes, not liver enzymes, are the ones needed to aid digestion and are administered with all snacks and meals. 4. Incorrect: Pancreatic enzymes must be taken within 30 minutes of eating. Keep in mind that the beads should not be crushed or chewed.

The nurse is preparing to administer magnesium sulfate IV to an alcoholic client with hypomagnesemia. Prior to the initiation of IV magnesium, which assessment data would be important for the nurse to document? 1. Liver function 2. Respiratory rate 3. Calcium levels 4. Deep Tendon Reflexes (DTRs) 5. Urinary output

2., 4., & 5 Correct: As you learned, magnesium acts like a sedative. Since we know that magnesium can cause respiratory depression, the nurse should always have a baseline respiratory assessment prior to initiating an infusion of magnesium. Muscle tone and DTRs can also become depressed, so a baseline assessment of DTRs would be very important. How is magnesium excreted? That's right! Through the kidneys. The nurse should always assess kidney function and urinary output prior to and during IV magnesium administration because of the risk of magnesium toxicity if it is being retained. 1. Incorrect: Magnesium administration does not impair liver function, so although the alcoholic client may have altered liver function, this is not an assessment that the nurse would be most concerned about related to magnesium administration. In fact, hypomagnesemia is a common problem in alcoholics which may require increasing foods high in magnesium or magnesium supplementation by PO or IV routes. 3. Incorrect: Magnesium levels are not influenced by calcium levels, so this is not an assessment that would be a priority for the nurse at this time.

What information on burn prevention strategies should the nurse include when providing an education program at a community center? 1. Have chimney professionally inspected every 5 years. 2. Clean the lint trap on the clothes dryer after each use. 3. Keep anything that can burn at least 1 foot (0.30 meters) away from space heaters. 4. Do not hold a child while holding a hot drink. 5. Home hot water heater should be set at a maximum of 120°F (48.8°C).

2., 4., & 5. Correct: Lint that accumulates in the lint trap of a dryer can cause a fire, so the lint trap should be cleaned after each use. A hot beverage can easily spill on a child by accident when trying to handle both the beverage and child at the same time. Home hot water heater should be set at a maximum of 120°F (48.8°C), especially when small children, the elderly, or diabetics are in the home. 1. Incorrect: A chimney should be professionally inspected every year prior to use. It should also be cleaned if necessary. 3. Incorrect: Space heaters need space at least three feet (0.91 meters) away from anything that can burn.

The nurse is preparing a teaching plan for a client newly diagnosed with fluid retention and heart failure. What should the nurse advise the client to avoid? 1. Broiled, fresh fish 2. Effervescent soluble medications 3. Seasoning with lemon pepper 4. Chicken noodle soup 5. Deli-ham sandwiches

2., 4., & 5. Correct: Think about fluid volume excess and heart failure. Things such as effervescent soluble medications and canned/processed foods should be avoided because they all contain a lot of sodium which increases fluid retention. Therefore, the chicken noodle soup and the cold cut deli-ham sandwiches should be avoided. 1. Incorrect: Fresh fish is a good, healthy selection that is low in sodium, which is what this client needs. Make sure to avoid smoked or cured fish/meats because these would have a higher sodium content. 3. Incorrect: Salt, as a seasoning, should be avoided because this would increase the fluid retention problem. However, a good alternative to salt for seasoning foods is to use lemon, lemon juice, and pepper. These are lower in sodium than salt.

A client, who received blunt chest trauma from an all-terrain vehicle accident, is admitted to the unit at 7 PM following insertion of a chest tube at 5 PM. The drainage collection chamber has 80 mL of drainage present upon arrival to the unit. Which assessment finding would be of concern to the nurse? 1. Continuous bubbling is occurring in the suction control chamber. 2. Intermittent bubbling is noted in the water seal chamber. 3. CDU is sitting upright on the bedside table with fluid levels as prescribed. 4. Slight fluctuations of water level in water seal chamber with respirations. 5. 190 mL of drainage noted in drainage collection chamber at 8 PM.

3. & 5. Correct: Do you see the problem with the bedside table? Yes! It's too high! The chest tube system should always be kept below the level of the chest to prevent backflow of drainage or air into the pleural space. You want to promote gravity drainage. The next problem that we see is excessive drainage. The chest tube was inserted at 5 PM and the client was admitted to the unit 2 hours later. The amount of drainage at upon arrival at 7 PM was 80 mL. At 8 PM, there was 190 mL of drainage. This is 110 ml of drainage in one hour. Drainage of 100 mL or greater any hour after the first hour is considered excessive. The healthcare provider would need to be notified of this amount of drainage. 1. Incorrect: Gentle, continuous bubbling in the suction control chamber is normal. 2. Incorrect: Intermittent bubbling in the water seal chamber when the client coughs, sneezes, or exhales is considered normal. 4. Incorrect: A slight rise and fall (fluctuation) of the water in the water seal chamber as the client breathes is called tidaling and is normal.

A client with a history of paranoid personality disorder is admitted to the hospital with extreme weight loss. Family states client has been refusing medications and food due to fears of being poisoned. What initial response by the nurse is most important? 1. "Tell me about your fears of being poisoned." 2. "No one is trying to poison your food or meds." 3. "You certainly are having scary thoughts." 4. "You are starving yourself needlessly."

3. CORRECT. The client is so fearful of being poisoned that physical harm has occurred secondary to personal starvation. The responsibility of the nurse is to address the client's fears and establish a trusting nurse/client relationship in order to meet the goal of helping the client feel safe enough to begin to eat. 1. INCORRECT. It is important not to focus on the client's perception of being poisoned by asking for information or clarification. Feeding into the delusion will reinforce that false reality for the client. This is an incorrect open-ended statement by the nurse. 2. INCORRECT. Even though the nurse is making an accurate statement, the client's perception of reality will negate anything that is stated by staff. The nurse is trying to refute what the client believes is true, which means the client will also distrust the nurse. Again the focus is on the poisoning instead of client's feelings. 4. INCORRECT. The client's fear and delusion about poisoning is strong enough to over-ride the pain of starvation. Such an ingrained thought would not be easily changed by this statement. Additionally, this comment by the nurse is belittling what the client assumes to be true, thus eliminating any chance for a trusting nurse/client interaction.

The nurse is educating a group of college students about early signs and symptoms of cancer. When explaining the mnemonic "C-A-U-T-I-O-N", the nurse explains the 'N' stands for what sign/symptom? 1. Nausea 2. Nipple drainage 3. Nagging cough 4. Nose bleeds

3. CORRECT. The mnemonic "C-A-U-T-I-O-N" represents an easy way to recall the seven early warning signs and symptoms of potential cancer. Each letter indicates a specific body alteration that should be reported to the primary healthcare provider. 'N' stands for a nagging cough or hoarseness. 1. INCORRECT. Though indigestion and difficulty swallowing are considered among the seven warning signs of cancer, nausea and vomiting are vague symptoms which can be attributed to a variety of disorders. These do not represent the "N" in 'CAUTION'. 2. INCORRECT. While any type of drainage from the breast should be reported to the primary healthcare provider, drainage is represented under the "U" for unusual discharge or bleeding. This is not the correct interpretation for the "N". 4. INCORRECT. A nose bleed could be the result of many factors, including clotting issues or even a dry environment. This symptom does not represent an early sign of cancer.

The nurse is teaching a group of female clients how to perform a self-breast exam. One client reports no family history of breast cancer and indicates disinterest in learning the technique. What is the most appropriate response by the nurse? 1. "You can ask your healthcare provider to do this with your yearly physical." 2. "If you have no family history of cancer, you won't need to worry about this." 3. "Self-breast exams may detect changes early enough for successful treatment." 4. "You have the right to refuse anything related to health because of client rights."

3. CORRECT. The nurse responds to this client's incorrect statement by presenting an accurate fact in a non-judgmental and open manner, allowing for further discussion about breast cancer facts. The nurse has a responsibility to provide the client important data about the topic of self-breast exams. I. INCORRECT. A breast exam completed only once a year is not often enough. Self-breast exams should be done monthly by both male and female clients. Some cancers are so aggressive that waiting a year could lead to a terminal diagnosis. 2. INCORRECT. This comment by the nurse is totally false. A family history of cancer is not a precursor to the occurrence of breast cancer. 4. INCORRECT. Though this closed-ended statement is accurate, the nurse has not provided the client with any information which could correct the client's misconceptions about breast-cancer.

An elderly client with partial and full-thickness burns has begun receiving fluids at 600 ml/hour, as determined by the Parkland (Consensus) Formula. Based on the assessment data for the first four hours, what should the nurse report to the primary healthcare provider? 1. The cardiovascular system is becoming seriously overloaded 2. The speed of the IV should be reduced since CVP is now normal 3. The changes in vital signs indicate an expected response to fluids 4. The client is deteriorating because of age and extent of the burns

3. CORRECT. The purpose of infusing large amounts of fluid into burn victims during the first 24 hours is to help maintain perfusion until the body's physiology returns to normal functioning. The serial vital signs indicate the cardiovascular system is stabilizing, as evidenced by pulse decreasing to the normal range while blood pressure increases. Though respirations are still slightly elevated, the client would likely be experiencing pain. Most importantly, the CVP (central venous pressure) has increased to the normal range, indicating the fluid replacement is adequate at this time. 1. INCORRECT. There is no evidence indicating possible cardiac overload. The client's vital signs are stabilizing and the central venous pressure (CVP) has returned to normal limits. 2. INCORRECT. When fluid replacement is calculated for burn clients, the amount is based on client weight in kilograms and total surface area burned. Those parameters do not change during the initial treatment. Therefore the amount of fluid needed during the first 24 hours remains unchanged until after that time frame, even if vital signs improve. 4. INCORRECT. The hourly data does not reflect deterioration. Vital signs are slowly returning to within normal range and there is no mention in the scenario about the extent of burns.

An confused elderly client is admitted with a diagnosis of malnutrition following a 30-pound (13.6 kg) weight loss in a month. The family requests insertion of Peg tube for enteral feedings, despite the client's advanced directives indicating "no life-prolonging measures". What is the most appropriate comment by the nurse to the family? 1. "Perhaps you could convince your parent to allow a Peg tube insertion." 2. "Maybe the client just needs family to prepare meals and help feed the client." 3. "The client completed an advanced directive form specifying what we may do." 4. "It is the client's right to refuse procedures not wanted."

3. CORRECT. This statement by the nurse provides an explanation of advanced directives as well as the fact the client has completed such a form. The focus is placed on the purpose of advanced directives and how medical personnel must abide by the client's wishes. The nurse has given the family a response which includes accurate knowledge as well as advocating for the client. 1. INCORRECT. Such a statement does not focus on the client's right to refuse life-extending procedures as noted in the advanced directives. The family is given false hope rather than a correct explanation regarding advanced directives and client rights. 2. INCORRECT. The nurse is attempting to refocus the family on the client's weight loss rather than the client's choices. The issue at this time is not the cause of the weight loss, but rather the client's right to refuse life-extending procedures as detailed in the advanced directives. 4. INCORRECT. Though this statement is accurate, it is abrupt and closed-ended. When addressing family, the nurse needs to remember that stress, fear and frustration can overwhelm judgment. The family may fear losing the client and the nurse's statement would not address those fears or the advanced directives.

A client returns to the room post appendectomy. In what position should the nurse place the client? 1. Sims' 2. Prone 3. Semi-fowler's 4. Right lateral

3. Correct: After any major abdominal surgery, the position of choice is to elevate the head of the bed 35-45 degrees. This will decrease pressure on the abdomen and suture line. 1. Incorrect: Sims' is a semi-prone position where the client assumes a posture halfway between lateral and prone. This is used for clients who need their airway protected. 2. Incorrect: Prone is not recommended. This will put more pressure on the suture line and abdomen. 4. Incorrect: Slightly side lying would be okay if the head of the bed was elevated to decrease abdominal and suture line pressure. The best position is semi-fowler's immediately post op.

What medication, given to help mature fetal lungs, does the nurse anticipate giving after admitting a client in preterm labor? 1. Magnesium sulfate 2. Terbutaline 3. Betamethasone 4. Nifedipine

3. Correct: Betamethasone, a steroid, is given IM to help the fetal lungs mature. 1. Incorrect: Magnesium Sulfate is given IV because it relaxes the uterus in an effort to stop contractions. 2. Incorrect: Terbutaline is given SQ because it relaxes the uterus in an effort to stop contractions. 4. Incorrect: Nifedipine is given PO because it relaxes the uterus in an effort to stop contractions.

The nurse is caring for a client diagnosed with heart failure who has developed pulmonary edema. Which finding best indicates that bumetanide is having a therapeutic effect? 1. Apical pulse 108/irregular. 2. Foamy sputum. 3. Urine output 175 mL for one hour. 4. Respiratory rate 28/min

3. Correct: Bumetanide is a diuretic that can be given IVP or continuous IV to provide rapid fluid removal. We know the medication is working because we have a good hourly urinary output. 1. Incorrect: The heart rate is still too fast and irregular. As excess fluid is removed, the heart rate should come down to a regular rate. 2. Incorrect: Pulmonary edema will cause the client to have a productive cough with pink, frothy (foamy) sputum. The presence of foamy sputum does not indicate that the medication has been effective. 4. Incorrect: The respiratory rate is too fast, so the pulmonary edema has not resolved. As fluid is pulled off the body, the respiratory rate should decrease.

A client in the intensive care unit who is on the ventilator, suddenly exhibits signs of decreased cardiac output. A quick assessment reveals that the client has cyanosis, absence of breath sounds on the right side, neck vein distention, and the trachea is deviating to the left. What initial emergency measure should the nurse expect to be performed? 1. Insertion of a chest tube in the 7th intercostal space 2. Immediate removal of client from the ventilator 3. Needle decompression in the right 2nd intercostal space 4. Emergency thoracentesis of the left lung

3. Correct: Did you recognize the signs of a tension pneumothorax? This client may have developed this because of a high PEEP level and/or compromised lung status combined with mechanical ventilation. Regardless of the cause, this is an emergency situation and the initial treatment involves the insertion of a large bore needle into the 2nd intercostal space, midclavicular line of the affected side. In this case, you should recognize that the absence of breath sounds on the right side indicate that the problem is on the right side. Needle decompression is done to release the pressure that is building up in the pleural space and causing the organs and vessels to be compressed. The mediastinal shift occurs toward the opposite (left) side. The client will most likely have a chest tube inserted on the right side, but the initial life saving measure for this would be the needle decompression. 1. Incorrect: What did you learn that is being removed when a chest tube is inserted low in the chest wall? That's right! Fluid. In this case, your clues were that the client was on mechanical ventilation and suddenly developed signs and symptoms of a tension pneumothorax. That means that air is accumulating and causing the problem. We need to remove this air that is compressing the vessels and causing decreased cardiac output. 2. Incorrect: This is a safety issue! Would you turn off a ventilator of a client that is needing this for ventilation ("artificial breathing")? No! You must deal with the problem and decompress the air that is accumulating in the pleural space. 4. Incorrect: The problem is on the right side. There is no need for a thoracentesis on the left side.

A child, admitted to the emergency department is noted to be drooling and has dysphagia. No cough is noted, and the child appears worse than the sound indicates. The parent states the child seemed "fine" when put to bed. History reveals that the child has not received some of the recommended immunizations. What should the nurse anticipate as part of the care for this child? 1. Placement in the lateral, supine position. 2. Prompt initiation of respiratory syncytial virus immune globulin. 3. Transfer to OR for placement of ET tube. 4. Oral dose of dexamethasone.

3. Correct: Did you recognize these symptoms as being characteristic of epiglottitis? Yes! It is considered a medical emergency in which there can be rapid progression to severe respiratory distress due to airway occlusion. An endotracheal tube (ET) may be needed, but it is best for the child to be in the OR where anesthesia can be administered, and an emergency trach can be performed if the airway is too occluded for the passage of the ET tube. 1. Incorrect: Did you pick the lateral, supine position because there is drooling present? Don't let that trick you here. The drooling occurs because of the degree of inflammation affecting the epiglottis. Because of the potential for rapid progression of respiratory distress, we want to promote ease of respirations, minimize agitation, and allow the child to be in the position of comfort. A great position is to be upright being comforted in the parent's arms. 2. Incorrect: Respiratory syncytial virus immune globulin is not used for the treatment of epiglottitis. 4. Incorrect: Although steroids may be used in the treatment of epiglottitis, you would never want to try to administer anything by mouth to this child who has drooling and dysphagia. The child can't swallow effectively! IV medication administration would be the route of choice in the acute period.

A client was admitted with reports of prolonged diarrhea. The client's admission potassium level was 3.3 mEq/L (3.3 mmol/L) and is receiving an IV of D5 ½ NS with 20 mEq KCL at 125 mL/hr. The UAP reports an 8 hour urinary output of 200 mL. The previous 8 hour urinary output was 250 ml. What should be the nurse's priority action? 1. Encourage the client to increase PO fluid intake. 2. Administer a supplemental PO dose of potassium. 3. Stop the IV potassium infusion. 4. Administer polystyrene sulfonate PO

3. Correct: First, you need to recall that potassium is excreted by the kidneys. If the kidneys are not working well, the serum potassium will go up! You always monitor the urinary output before and during IV potassium administration. Since the urine output has decreased below 30 mL/hr, we know that the urinary output is not adequate. Therefore, the client could start retaining too much potassium. The priority action would be to stop the infusion and then follow this action by notifying the healthcare provider. 1. Incorrect: You may have picked up on the decreased output and thought that you could increase PO fluid intake to increase output. However, the priority action would be to first stop the potassium infusion until the urinary output is adequate. This is a safety issue. 2. Incorrect: We do not want to administer any more potassium to this client. The urine output is not adequate and the client could be retaining too much potassium. 4. Incorrect: Polystyrene sulfonate (Kayexalate®) is used as a treatment for clients with known hyperkalemia. We are trying to prevent this client from becoming hyperkalemic by stopping the IV potassium infusion as the urine output has decreased.

A child is admitted in a sickle cell crisis. What treatment should the nurse anticipate being most helpful in reducing the painful crisis? 1. Antibiotics 2. Oxygen 3. Hydration 4. Bedrest

3. Correct: Hydration is crucial with a sickle cell crisis. It helps minimize the vaso-occlusive process that is causing the pain as it pushes the sickled cells apart, allowing them to flow through the vessels more freely. 1. Incorrect: Antibiotics may be needed if an infection is present, but this is not the most beneficial in reducing the painful crisis. 2. Incorrect: Oxygen does not reverse the sickling process and does not help improve circulation of the sickled cells that is occluding the vessels and causing the pain. It may be given to help improve hypoxia and prevent further sickling. 4. Incorrect: Bedrest is needed to help conserve O2. In addition, activity can lead to increased pain. However, it is not the treatment that is most helpful in reducing the blockage from the sickled cells that leads to the painful episodes.

A nurse on a surgical unit is assigned a client who had a total thyroidectomy 3 days ago. As the nurse enters the room which nursing assessment is the priority for this client? 1. Eating a soft diet. 2. Positioned at 15 degrees in bed. 3. States hands are tingling. 4. Expresses frontal neck pain level of 5 out of 10.

3. Correct: Hypocalcemia is a severe complication of a thyroidectomy due to damage to the parathyroid. The negative feedback of a low parathyroid hormone (PTH) results in a decrease in serum calcium. PTH regulates the amount of calcium levels in the blood. Symptoms of hypocalcemia include numbness, and tingling on the extremities and face. As the calcium levels decrease the client may present with tetany and spasm of the larynx. 1. Incorrect: The postoperative diet for a client post thyroidectomy begins with ice chips and progresses to a liquid diet for approximately 2 days, and then a soft diet. The last dietary step is diet as tolerated. As the diets progress, the nurse should assess the ability of the client to swallow and changes in the voice such as hoarseness may indicate swelling. 2. Incorrect: On the 3rd postoperative day the client's bed can be positioned at 15 degrees. The nurse should monitor the client's airway for any problems with breathing. If the client experiences any airway difficulty, the nurse should change the bed to a semifowlers or high fowlers position. 4. Incorrect: Whenever a client problem such as pain is identified, a nursing intervention must address the problem. The priority intervention is to address the assessment of the client stating that their hands are tingling. Symptoms of hypocalcemia include numbness and tingling of the extremities and face.

A nurse is preparing to obtain vital signs on a 2 year old. What should the nurse consider when preparing to perform this task? 1. The blood pressure should be obtained first to get an accurate reading. 2. Count the RR and HR for 30 seconds to avoid prolonged disturbance. 3. If the child becomes upset, record the behavior with the measurements. 4. The axillary route is the most reliable route for checking the temperature.

3. Correct: Infants, toddlers, and young children often become anxious or upset during procedures, such as vital sign measurement, and we know that this activity could affect the vital sign results. Nurses or other healthcare providers would need this information to consider when evaluating the results. 1. Incorrect: We want to obtain the least invasive vital sign first. Start with observation before touching the child. What vital sign can you get by observing? Yes! Respirations! 2. Incorrect: In infants and toddlers, it is important to count the RR and HR for one full minute because of irregularities due to their immature nervous system regulation. 4. Incorrect: The rectal route is considered the most reliable route for assessing the temperature in infants and children.

The nurse is caring for a surgical client who developed a pulmonary embolus (PE). Which diagnostic test would be the most sensitive for providing a definitive diagnosis for a PE? 1. D-dimer 2. Pulmonary function test 3. Pulmonary angiography 4. Chest X-ray

3. Correct: Pulmonary angiography is the most sensitive and specific test for a PE. However, since it is very expensive and invasive, the computerized tomography angiogram (CTA) is the most frequently used test to diagnose a PE. 1. Incorrect: The D-dimer will be increased if a pulmonary embolism is present. However, since the client had surgery, you know that the D-dimer will be increased already because it simply tells if a clot is located anywhere in the body. It is not specific to clots in the lungs. 2. Incorrect: Pulmonary function tests provide information about how well the lungs are working. Various aspects such as lung volume, lung capacity, gas exchange, and rates of air flow can be determined. Although these help diagnose a lung problem, they are not specific to diagnosing a pulmonary embolism. 4. Incorrect: Chest x-rays are not very sensitive nor specific for diagnosing pulmonary embolisms in clients. Although clots do not show up on the x-ray, other findings that are suggestive of a PE may be found to indicate the need for further testing.

Which laboratory test should be assessed by the nurse prior to administering radioactive iodine (RAI) to a female client? 1. Thyroid Scan 2. Serum calcium 3. Pregnancy test 4. Metanephrine test

3. Correct: RAI crosses the placenta and will affect the development of the fetus. If RAI is administered to a client who is pregnant, the fetus can experience mental retardation, hypothyroidism, and develop increased cancer risk. It is imperative that a pregnancy test should be prescribed prior to administering RAI. RAI should not be administered to a client who has a positive pregnancy test. 1. Incorrect: A thyroid scan is prescribed to evaluate the function and size and shape of the thyroid gland. This scan can identify the amount of thyroid hormone the thyroid is producing (hyperthyroidism) Also the thyroid scan will evaluated for the presence of thyroid nodules. Prior to the administration of RAI, the female client should have a pregnancy test prescribed. This test will not identify if the client is pregnant. 2. Incorrect: A calcium test is prescribed to analyze the calcium level in conditions affecting nerves, parathyroid, kidney, and thyroid. Since this test does not identify whether a client is pregnant, a calcium test result is not related to whether the nurse administers the RAI. 4. Incorrect: The metanephrine test measures the amount of metanephrine in urine. The breakdown of catecholamines results in metanephrine. The catecholamines are epinephrine, norepinephrine and dopamine. The female client should be evaluated for pregnancy prior to administrating RAI.

What action by the nurse, who is administering platelets to a client, would require the charge nurse to intervene? 1. Verifies prescription for platelet transfusion. 2. Confirm client has provided informed consent. 3. Hangs platelets immediately upon arrival from blood bank refrigerator. 4. Infuse platelets with normal saline solution.

3. Correct: The charge nurse needs to intervene here. NEVER infuse cold platelets, because the spleen will reject them if they are cold and not absorb them. So, room temperature for your platelets or they will do you no good. 1. Incorrect: The charge nurse does not need to intervene here. The nurse should check for a prescription to administer platelets. 2. Incorrect: This is also a correct action by the nurse, so there is no need for the charge nurse to intervene. 4. Incorrect: Normal saline is the acceptable fluid to hang with blood and blood products, so the charge nurse does not need to stop the nurse.

The nurse has initiated discharge instructions for a client diagnosed with glomerulonephritis. What statement by the client would indicate to the nurse that further teaching is needed? 1. "I will have protein in my urine for several months." 2. "My urinary output will increase in 1 to 3 weeks." 3. "I should keep a record of the headaches I experience over 3 months." 4. "I should notify my primary healthcare provider if my urinary output decreases."

3. Correct: The client should report headaches sooner than 3 months to their primary healthcare provider. The headaches are related to the fluid retention due to the decreased filtration of the glomerulus. The retention of fluid will result in hypertension. This client will require additional discharge instructions. 1. Incorrect: This is a correct statement by the client. Due to the damage to the glomerulus, the kidneys will leak protein. The damage to the kidneys may take several months to heal properly. The kidneys will leak protein for several months. 2. Incorrect: When the glomeruli are restored, the kidneys will begin to diuresis. The diuresis usually starts in 1 to 3 weeks. This is a correct discharge statement by the client. 4. Incorrect: The glomerulus and the surrounding Bowman capsule create a renal corpuscle. The glomerulus filtrates the blood which results in urine formation.

What should the nurse do after administering a chemotherapeutic drug intravenously (IV) to a client in the outpatient infusion unit? 1. Hang a 250 mL normal saline (NS) bag to flush the IV line. 2. Wear shoe covers during disposal of the drug. 3. Place the IV bag and tubing into a chemotherapy waste container. 4. Disposal of personal protective equipment (PPE) in a biohazardous container.

3. Correct: The disposable items such as the IV bag and tubing should remain intact and be disposed of in a securely sealed chemotherapy waste container. Tubing should never be disconnected from an IV bag containing a hazardous drug because of the risk of splashing. 1. Incorrect: The IV line does not have to be flushed with that much fluid. Usually, a central venous access device (CVAD) is flushed with 10 mL NS and a heparin solution if appropriate for the client and the device . For a peripheral line 2-5 mL NS is sufficient.2. Incorrect: Shoe covers are not needed. Shoe covers are used when there is a chance of walking in contamination such as blood spilled on floor.4. Incorrect: PPE (including double gloves, goggles, and protective gown) should be worn for all activities associated with chemotherapeutic drug administration. PPE used in chemotherapy drug administration should be disposed of in chemotherapy—not biohazardous—waste receptacle.

While preparing chemotherapy, the nurse accidently punctures the bag, spilling the solution on the floor. After activating the emergency spill protocol, what action should the nurse take first? 1. Place absorbent pads and absorbent powder over the spill. 2. Apply chemotherapy approved personal protective gown and gloves. 3. Obtain the proper spill kit for the specific chemotherapy drug used. 4. Post the "Caution-Chemo Spill" sign outside the room door.

3. Correct: The first step in handling an accidental chemotherapy spill is to obtain the specific spill kit for that type of drug since each medication may require a different cleanup protocol. 1. Incorrect: This is step 4: Each kit includes special absorbent pads and usually a powder designed to solidify liquid, making it easier to contain. 2. Incorrect: This is step 3. The nurse should then apply all the personal protective equipment (PPE) in the kit, including gown, mask, goggles, shoe covers and two pairs of special chemo gloves. Chemo drugs are deadly, whether ingested, injected or inhaled. The nurse needs to avoid any potential contact with the drug. 4. Incorrect: This is step 2. Inside the kit is the "Caution-Chemo Spill" sign, which must immediately be posted outside the room and the door closed to contain the spill. Step 5: Once the spill is safely cleaned up, the nurse must report the spill to the facility supervisor and the client's primary healthcare provider in addition to completing an incident report.

The nurse is teaching a client who has been prescribed peritoneal dialysis. What statement by the client indicates to the nurse that teaching was successful? 1. "I need to decrease protein in my diet since my kidneys no longer work." 2. "Heating the dialysate in the microwave for 30 seconds will prevent abdominal cramping." 3. "I will notify my primary healthcare provider if the peritoneal drainage is cloudy." 4. "The automated peritoneal dialysis (APD) cycler is used every few hours during the day."

3. Correct: The number 1 complication of peritoneal dialysis is infection. So, the client does need to monitor the drainage, which should be clear or straw-colored. If it is cloudy, that indicates infection and the primary healthcare provider should be notified. 1. Incorrect: This client needs to increase protein intake because the client is losing protein through the peritoneal membrane with each exchange. 2. Incorect: Do NOT put dialysate in the microwave! We don't want to burn the peritoneum. Take it out of the refrigerator and allow to warm to body temperature. 4. Incorrect: The APD cycler does exchanges automatically throughout the night while the client is sleeping.

What is the most important action for the nurse to take prior to a client having a liver biopsy? 1. Make certain the consent has been signed. 2. Obtain vital signs. 3. Check clotting study results. 4. Position client supine with right arm above head.

3. Correct: This is a priority question. All actions should be done by the nurse, however, the nurse better check the clotting study results. The client could hemorrhage if the clotting factors are too messed up. 1. Incorrect: Yes, the consent must be signed, but what is more life saving? Checking the clotting factor results. 2. Incorrect: Yes, the nurse will need to obtain pre-procedure vital signs. However, the procedure may not be done if the clotting study results are bad. 4. Incorrect: Yes, the client will need to be positioned so that the primary healthcare provider has access to the liver. But again, this is not the priority.

The client has been instructed on crutch safety. The nurse identifies that further teaching is needed when the client makes which statement? 1. "The crutches are adjusted according to my height." 2. "I will support my weight on the hand grips when not walking." 3. "I plan to place my affected leg on the step first when ascending stairs." 4. "I will position the crutches 1 -2 inches below the axilla when walking with crutches."

3. Correct: This statement is incorrect and further client teaching is needed. When going up stairs, the client should lead with the unaffected leg. The unaffected leg will provide the support required to then move the affected leg to the step. 1. Incorrect: The crutches should be adjusted according to the client's height and arm length. The top of the crutches should be approximately 1 to 2 inches under the axilla. The hips should be even with the hand grips. Also, the crutch length should be measured from the client's axilla to approximately 6 inches in front of the toe. This is a true statement. 2. Incorrect: If the weight is supported by placing the top of the crutches against the axilla, then brachial nerve damage will occur. To prevent the damage to the brachial nerve the hands rest on the hand grips when resting. This is a correct statement by the client. 4. Incorrect: To prevent damage to the brachial nerve, the client should position the crutches 1 to 2 inches below the axilla when walking with crutches. With the shoulders relaxed the client should be able to also position 2 finger widths between the axilla and the crutch pads.

While performing wound care to a donor skin graft site, the nurse notes some scabbing around the edges and a dark collection of blood. What is the nurse's next action? 1. Leave the scabbing area alone and apply extra ointment. 2. Notify the primary healthcare provider. 3. Gently remove the debris and re-dress the wound. 4. Apply skin softening lotion for 3 hours and then re-dress.

3. Correct: What likes to live in the scabs and dried blood? Bacteria. That is why it is important to remove the debris to prevent infection. 1. Incorrect: This is not appropriate because bacteria is in the scabbing area and ointment would trap it, enhance reproduction of the germs, and increase infection. 2. Incorrect: There is no need to notify primary healthcare provider at this time. This is not the best option for the nurse to fix the problem. 4. Incorrect: We don't put lotion in the wound because this would cause infection of the wound.

The nurse is providing stump care discharge instructions to the client with a right below-the-knee amputation (BKA). The client responds, "What is the purpose of the compression sock on my stump?" Which statement by the nurse is appropriate? 1. "The compression sock on the stump will increase your balance when crutch walking." 2. "Phantom limb pain will decrease by applying the compression sock tightly around the stump." 3. "A compression sock is applied to shape the stump smaller and rounder on the bottom." 4. "The application of a compression sock will decrease the risk of the incidence of deep vein thrombosis (DVT)."

3. Correct: Wrapping the stump with an ace bandage will assist in configuring the stump into a cone shape. The cone shape is smaller and rounded on the bottom. The cone shaped stump will result in the stump fitting easier into the prosthesis. 1. Incorrect: The compression sock will not increase the client's balance when crutch walking. The compression sock will assist in shaping the stump. 2. Incorrect: This is an incorrect statement. The nurse's interventions to decrease phantom pain would include diversional activity and administering the prescribed analgesic. 4. Incorrect: Applying a compression sock to the right stump is not an appropriate intervention to decrease the risk of a DVT. The risk for a DVT after surgery is increased in the left leg. Interventions to decrease a DVT are to move the extremities frequently and increasing fluid intake.

The nurse is teaching a group of clients in cardiac rehabilitation how blood flows through the heart. What information should the nurse include? 1. Deoxygenated blood enters the heart through the pulmonary vein. 2. Blood flows from the right atrium through the mitral valve to the right ventricle. 3. The right ventricle pumps the blood to the lungs via the pulmonary artery where the blood becomes oxygenated. 4. From the lungs, oxygenated blood goes to the left atrium via the pulmonary vein, then to the left ventricle. 5. The right ventricle pumps the blood out through the aorta to the body.

3., & 4. Correct: These are true statements. The right ventricle pumps the blood to the lungs via the pulmonary artery where the blood becomes oxygenated. From the lungs, oxygenated blood goes to the left atrium via the pulmonary vein, then to the left ventricle. 1. Incorrect: Deoxygenated blood comes from the body to the heart via the superior and inferior vena cava. 2. Incorrect: Blood flows from the right atrium through the tricuspid valve to the right ventricle. 5. Incorrect: The left ventricle pumps the blood out through the aorta to the body.

A client is diagnosed with a duodenal ulcer due to Helicobacter pylori (H Pylori). In addition to antibiotic therapy, the nurse anticipates that the client will also receive what class of pharmacologic agents? 1. Miotic inhibitor 2. Serotonin antagonist 3. H2 antagonist 4. Acetylsalicyclic acid 5. Proton pump inhibitor

3., & 5. Correct: H2 antagonist or receptor blockers are used to decrease excess stomach acid seen with ulcers. Antisecretory agents like proton pump inhibitors are indicated for the treatment of peptic ulcer disease. Antisecretory agents decrease the secretion of gastric acids. Protein pump inhibitors, a combination of antibiotics and bismuth salts are most commonly used for treatment of H Pylori. 1. Incorrect: Mitotic inhibitors are chemotherapeutic agents that are indicated for the treatment of malignancies and cancerous cells. They are most often used in combination chemotherapy regimens to enhance the overall cytotoxic effect. 2. Incorrect: Serotonin antagonists are antiemetic agents that are indicated for the treatment of nausea and vomiting. Serotonin antagonists block the serotonin receptor sites located throughout the body responsible for the mediation of nausea and vomiting. 4. Incorrect: Acetylsalicylic acid is a non narcotic analgesic that inhibits the cox-2 protective mechanisms to the gastric mucosa. This could make the ulcer worse. Clients are advised to avoid the use of NSAIDs and acetylsalicylic acid due to increased bleeding potential.

What interventions should the nurse include when caring for a client who is receiving total parenteral nutrition (TPN)? 1. Change tubing and filter every 48 hours. 2. Monitor IV drip rate hourly. 3. Compare new bag with prescription prior to infusing. 4. Weigh weekly. 5. Cover TPN with dark bag. 6. Check urine for protein.

3., & 5. Correct: Remember safety and that TPN is a medication. You must make sure that what is in the bag is what was prescribed, so double check the bag against the prescription. Cover the IV bag with a dark bag to prevent chemical breakdown. 1. Incorrect: The IV tubing and filter must be changed with each new bag. Remember: A bag cannot hang more than 24 hours. 2. Incorrect: TPN must be placed on an IV pump. Relying on calculating to maintain a drip rate is dangerous. The client could get too much TPN too fast without having it on a pump at the prescribed rate per hour. This is a safety issue. 4. Incorrect: The client should be weighed daily. We want to make sure the client is not losing weight while on TPN. They should be maintaining or gaining weight. 6. Incorrect: Monitor urine for glucose and ketones. The only way protein will be in the urine is if the kidneys are damaged.

How would the nurse interpret this client's Arterial Blood Gas (ABG) results? pH 7.35 PaCO2 30 mm Hg Bicarb 19 mEq/liter PaO2 89 mm Hg SaO2 90% 1. Respiratory acidosis 2. Respiratory alkalosis 3. Metabolic acidosis 4. Metabolic alkalosis 5. Uncompensated 6. Partially compensated 7. Fully compensated

3., & 7. Correct. These ABG values indicate metabolic acidosis. The pH is normal, but it is on the acidosis side of normal at 7.35. Now, which other chemical says acidosis? Look at the bicarb; the bicarb is low, indicating acidosis so there's your match! The bicarb matches the pH. What chemical problem does the bicarb relate to - respiratory or metabolic? It's metabolic. Metabolic acidosis. Has compensation begun? Yes. The lungs are compensating for the metabolic acidosis by getting rid of CO2, which is an acid. Therefore, the PaCO2 is below the normal range of 35-45. Since the pH is normal, full compensation has occurred. 1. Incorrect. For this problem to indicate respiratory acidosis, the pH would need to be less than 7.35 (or if fully compensated, the pH would be less than 7.40 to be on the acidotic side of normal) and the CO2 would need to be greater than 45. In this problem, we see the CO2 has been blown off to help get rid of the acid. 2. Incorrect. This is not a respiratory problem. The lung chemical, carbon dioxide does not match the acidotic pH. The pH indicates acidosis, not alkalosis. 4. Incorrect. Metabolic alkalosis would have pH greater than 7.45 (or if fully compensated, the pH would be greater than 7.40 to be on the alkalotic side of normal) and a Bicarb level greater than 26. The pH is on the acidosis side of normal, and the bicarb (metabolic chemical) indicates acidosis here. The lungs have compensated by bringing down the CO2 level to decrerase the acidotic state. 5. Incorrect. The pH is normal even though the PaCO2 and the bicarb values are abnormal so compensation has occurred. The pH would be abnormal and the PaCO2 would be normal if compensation had not begun. This client has fully compensated. 6. Incorrect. The pH is normal even though the PaCO2 and the bicarb values are abnormal so compensation has occurred. With partial compensation, the pH, PaCO2, and bicarb would all be abnormal. This client has fully compensated.

Which intervention would the nurse include when planning care for a client who has increased intracranial pressure (IICP)? 1. Place client supine. 2. Hyperextend head to maintain airway. 3. Maintain body temperature below 100.4 F (38 C). 4. Cluster nursing care. 5. Monitor vital signs for Cushing's Triad. 6. Limit suctioning.

3., 5., & 6. Correct. The goal of treatment is to relieve the IICP by reducing cerebral edema, reducing the amount of cerebrospinal fluid, or reducing the blood volume in the brain, We also have to maintain cerebral perfusion. An increased temperature will increase cerebral edema, which will increase ICP. Monitor vital signs for Cushing's Triad. This is systolic hypertension with a widening pulse pressure, a slow, full and bounding pule, and irregular respirations. Limit suctioning and coughing as these can make the client's ICP go up. 1. Incorrect. It is standard practice to elevate the head of the bed. ICP varies with position. 2. Incorrect. The head should be in midline or neutral position so that the jugular veins can drain. 4. Incorrect. Nursing interventions should be spaced. Anytime you do something to your client, their ICP is going to go up.

A client is scheduled to be admitted to the surgical unit post total laryngectomy. What nursing intervention should the nurse include in the plan of care? 1. Position left-side lying, supine. 2. Place on clear liquid diet after peristalsis returns. 3. Monitor tracheostomy for pulsations with heart beat. 4. Provide mouth care every 2 hours. 5. Maintain a humidified environment.

3.,4., & 5. Correct: If a client's trach is pulsating with the heartbeat, you need to notify the primary healthcare provider immediately, as this could lead to rupture of the innominate artery. Frequent mouth care will decrease the bacterial count in the mouth. We are trying to prevent pneumonia. When breathing in and out through a trach, the client will not be able to warm, filter, and humidify the air. The air is really dry, so it irritates the trach. That is why when the client first gets the trach it has a lot of secretions. A humidified environment will help. 1. Incorrect: Where is the surgery? At the neck. Swelling! So place mid-fowlers, head of bed 35-45 degrees. 2. Incorrect: Peristalsis can disrupt the suture line. NG tube feedings will be provided to protect the suture line.

An elderly client arrives at the emergency room reporting a severe headache and blurred vision. The client indicates having awakened this morning with flu-like symptoms including nausea, vomiting and dizziness. The nurse notes the client appears very weak with shortness of breath and dark cherry red lips. Based on assessment findings, what life-threatening problem does the nurse expect? 1. Guillian Barre 2. Severe dehydration 3. Advanced influenza 4. Carbon monoxide poisoning

4. CORRECT. Carbon monoxide is a colorless, odorless, tasteless gas which permeates the blood stream, displacing the oxygen in hemoglobin. Symptoms are often confused with other illnesses, such as the flu. Assuming exposure is not fatal, the client may also experience extreme weakness, dizziness and blurred vision with confusion. Additionally, the carbon monoxide will cause lips and skin to become red in color. Without treatment, the client will die. 1. INCORRECT. Guillian-Barre is a muscle disorder occurring when the immune system attacks peripheral nerves, destroying the surrounding myelin sheath. The damage can develop over hours or days, but will take months to resolve. The client experiences severe weakness, drooping of the eye muscles and pain or tingling in hands and feet. The client also develops paresthesia and paralysis, which was not reported as symptoms in the scenario. Of major concern would be paralysis of the respiratory muscles. 2. INCORRECT. Although the client reported nausea and vomiting, there are no assessment findings in the scenario to corroborate severe dehydration. 3. INCORRECT. The client has reported flu-like symptoms, such as dizziness, nausea and vomiting along with headache. However, additional reported symptoms like blurred vision suggest a different problem

An adolescent is admitted to the psychiatric unit following a repeat suicide attempt. What is the nurse's priority action? 1. Have staff check on client once every hour. 2. Ask client to explain why suicide was a choice. 3. Place client in quiet seclusion with lights off. 4. Assign a staff member to stay with the client.

4. CORRECT. The client is newly admitted following a repeat suicide attempt and therefore safety is the priority issue. The client should not be left alone, even when using the bathroom, until the primary healthcare provider determines the risk of suicide has abated. 1. INCORRECT. A suicidal client is never left unattended immediately after admission. Checking the client just once an hour increases the potential risk for another suicide attempt. 2. INCORRECT. Demanding an explanation of the client is not an appropriate nursing action and is non-therapeutic. Some clients may be unable to provide an answer while others may be unwilling to discuss the situation with the nurse. This does not focus on client safety. 3. INCORRECT. A suicidal client should not be placed in seclusion, and definitely not in a darkened environment. Such actions do not provide a safe environment for the client and may increase the risk for another suicide attempt.

A client has responded positively to a series of electroconvulsive treatments (ECT), but reports concerns about on-going memory loss. What is the most appropriate response by the nurse? 1. "It's only been a couple weeks so don't worry." 2. "Are you afraid your memory will not return?" 3. "I will ask the psychiatrist to come talk with you." 4. "You seem very concerned about your memory."

4. CORRECT. The nurse/client relationship is collaborative and nonjudgmental with the goal of facilitating the client's emotional growth. Open-ended statements or questions encourage the client to express feelings and continue verbalizing. This comment by the nurse is open-ended and acknowledges the client's concerns. 1. INCORRECT. Although the nurse has indicated the time frame for memory loss would be greater than two weeks, telling the client not to worry denies the client's right to express feelings. 2. INCORRECT. This question is closed-ended because the client can respond with yes or no. Also, the nurse has suggested feelings of fear, instead of letting the client identify a specific emotion. 3. INCORRECT. It is not appropriate for the nurse to transfer care of the client to another individual currently. Instead, first the nurse should address the client's concerns directly and immediately.

A client is admitted with a diagnosis of bacterial meningitis. Which action should the nurse initiate first? 1. Darken room. 2. Provide sponge bath for fever of 102 F (38.8 C). 3. Pad side rails. 4. Place on Droplet precautions

4. Correct. Bacterial meningitis is transmitted through the respiratory system. According to the Center of Disease Control (CDC), clients with bacterial meningitis should be placed on "Droplet Precautions". 1. Incorrect. Darkening the room will help comfort the client with photophobia. However, placing the client on droplet precautions will take priority. 2. Incorrect. The nurse will give a sponge bath to cool the client with a fever, but again, the priority is infection control. 3. Incorrect. The client with meningitis is at risk for seizures, but first place on droplet precautions. You must protect yourself and others first.

A nurse is caring for a client with fat embolus syndrome (FES). Which data would support the nurse's assessment that the FES has resolved? 1. Respirations - 24. 2. Oxygen saturation - 94%. 3. Arterial blood gas - pH 7.34. 4. No infiltrates noted on chest x-ray.

4. Correct: A fat embolism is caused by droplets of bone marrow fat that is released into the venous system. The droplets may lodge in the lungs. An x-ray of the lungs with the bone marrow fat will have a "snowstorm" appearance. A chest x-ray that does not identify any filtrates and does not have a "snowstorm" appearance is indicative the fat embolus is decreasing in size or completely resolved. 1. Incorrect: A respiratory rate of 24 is not within the normal range of respirations for an adult. If FES has resolved, you would expect the respiratory rate to be normal. 2. Incorrect: Oxygen saturation is the percentage of hemoglobin saturated with oxygen. A oxygen saturation value of 94% is not within the normal range of 95% to 100%. If FES has resolved, you would expect the oxygen saturation percentage to be normal. 3 Incorrect: The normal pH arterial blood gas range is 7.35-7.45. The client's pH level of 7.34 is not within the normal pH range. It is acidotic. The body regulates the pH level by changing the body's CO2, bicarbonate, oxygen levels. This lab value is not reflective of the resolution of a FES.

What immediate action should the occupational health nurse take once flames have been extinguished from a burned victim? 1. Remove jewelry. 2. Wrap in a clean blanket. 3. Cover burns with clean, dry cloth. 4. Briefly soak burned area in cool water.

4. Correct: Although all options are correct, the priority is to stop the burning process. Just putting out the flames is not enough to stop the burning process. You need to apply cool water briefly (no more than 10 minutes) to soak the burn area. Any longer can cause extensive heat loss. 1. Incorrect: Removing jewelry is important but stop the burning process first. Swelling occurs with burns, so jewlrey must be removed or you will not get it off. This can result in constriction of the extemity. Additionally, metal burns. 2. Incorrect: Wrapping the client in a clean or preferably a sterile blanket will help to hold in body heat. Remember, they have lost skin, the number one way to hold in body heat. 3. Incorrect: Applying a clean, dry cloth to the burn area will help prevent infection, but the priority is to stop the burning process.

A nursing instructor is presenting a discussion on nephrotoxic medications? Which class of medications would the instructor discuss? 1. Opioids 2. Antidiabetic 3. Corticosteroids 4. Aminoglycoside

4. Correct: Aminoglycoside antibiotics are nephrotoxic. Nephrotoxic medications can cause damage to the kidneys. Examples of aminoglycoside antibiotics are tobramycin, gentamicin, streptomycin, and paromomycin. Clients with kidney damage should not be prescribed aminoglycoside antibiotics. 1. Incorrect: Opioid medications are not classified as nephrotoxic. 2. Incorrect: Antidiabetic medications are not classified as nephrotoxic. 3. Incorrect: Corticosteroid medications are not classified as nephrotoxic.

A client is admitted to the emergency room with an open fracture of the left tibia which has been temporarily splinted. Which nursing intervention would the nurse implement? 1. Physically reduce the fracture. 2. Externally rotate the left leg. 3. Position the bed into a high Fowler's position. 4. Cover the fractured site with a sterile dressing.

4. Correct: An open fracture is when the bone has broken the skin and underlying soft tissue, and the bone is protruding from the wound. The nurse should cover the fracture site with a sterile dressing to prevent contamination of deeper tissues. 1. Incorrect: The leg was splinted as a temporary emergency intervention. Upon arrival in the emergency room, the fracture should not be reduced by the nurse. Once the skin has been broken at the fracture site, the wound is a portal of entry for contaminants. A surgical procedure is performed to clean the wound and the bone. 2. Incorrect: If the nurse externally rotates the left leg, there is an increased risk of additional trauma to the tissues from the movement of the fracture bone. Also, there is a risk of the bone slipping into the wound from the external rotation. 3. Incorrect: Placing the client in high Fowlers position is not an appropriate intervention. The fractured site and/or limb should be elevated.

The nurse notes continuous bubbling in the water seal chamber of the chest tube system. What should be the nurse's initial action? 1. Clamp the chest tube closest to the chest wall. 2. Increase the water level in the water seal chamber. 3. Have the client take a deep breath and do valsalva maneuver. 4. Notify the healthcare provider.

4. Correct: Continuous bubbling in the water seal chamber indicates that there is an air leak in the system. The healthcare provider should be notified. The healthcare provider may prescribe for the tube to be clamped at intervals along the tube for only a few seconds to determine the location of the air leak, but clamping of the tube should never be done without a prescription. 1. Incorrect: Clamping of the tube should never be done without a prescription. Clamping a chest tube can lead to a tension pneumothorax, which can be a life-threatening situation. 2. Incorrect: Increasing the water level in the water seal chamber will not help the air leak problem. Also, the levels of water for the water seal chamber and the suction control chamber are prescribed by the healthcare provider and should be maintained at the prescribed levels. 3. Incorrect: Will taking a deep breath and performing the Valsalva fix an air leak in the tube? No. So you need to notify the healthcare provider. Later, when it is time for the chest tube to be removed, you should instruct the client to take in a deep breath and do valsalva maneuver, but doing that in this situation will not help fix the problem.

A client is admitted to the ICU with diabetes insipidus following a head injury. Which finding would the nurse anticipate in this client? 1. Low serum hematocrit 2. High serum glucose 3. High urine protein 4. Low urine specific gravity

4. Correct: Diabetes insipidus is a condition that results from decreased ADH production. Therefore, the client will be diuresing large volumes of water which leads to a fluid volume deficit. We worry about shock in these clients. Keep in mind that concentrated makes #s go up and dilute makes #s go down in reference to specific gravity, sodium, and hematocrit. Here, the urine is very dilute which means the urine specific gravity will be low. 1. Incorrect: As the client loses volume through the kidneys, the blood (serum) will become very concentrated. Therefore, you would expect the hematocrit to be high, not low. 2. Incorrect: Don't let the name diabetes insipidus trick you into thinking it affects the glucose level. It is an ADH problem, not a glucose problem. We are worried about fluid volume deficit here. 3. Incorrect: You do not expect to see protein in the urine in DI. In fact, protein is not seen in the urine unless there is a kidney problem. This is an ADH problem, not a kidney problem. You are worried about a large amount of water loss with this client.

A client, who receives hemodialysis three times a week, has been placed on a fluid restriction of 1000 mL/day. What is the nurse's best action when the client is seen drinking a 12 ounce (360 mL) soft drink? 1. Take the soft drink away from the client. 2. Document the client is noncompliant. 3. Notify dietary to no longer send beverages with food trays. 4. Reinforce the importance of the fluid restriction with the client.

4. Correct: Educate the client on appropriate choices and lifestyle changes that are necessary to manage the client's condition. 1. Incorrect: It is ultimately the client's choice to drink a soft drink. Education will help the client make an informed decision. 2. Incorrect: The nurse should reinforce the purpose of fluid restriction, not just document noncompliance. Perhaps it is just a matter of lack of knowledge. 3. Incorrect: The client has the right to make choices and the nurse should reinforce teaching.

A 5 year old girl is upset and saying she is to blame for her brother getting hit by a car on his bike because she was mad at him earlier and wanted to hit him. What does the nurse recognize this type thinking to be in a child? 1. Abstract 2. Egocentric 3. Animism 4. Magical

4. Correct: Magical thinking is common in young children and is the belief that the world around them can be influenced or impacted by their own thoughts, desires, or wishes. Therefore, when something happens that is related to their thoughts, the child may perceive that it occurred because of those thoughts. This child may have connected the thoughts of being upset with the brother and the desire to hit him with the aspect of the car hitting him later. Other times, this magical thinking may be linked to a desire to make positive things happen by their thoughts. The interesting part about magical thinking is that young children may believe that they can make things or events in life be anything or anyway they want them to be. 1. Incorrect: Abstract thinking is the ability to think about objects, ideas, and principles that do not physically exist. It is the ability to understand relationships, critically think or reason, and think symbolically using a symbol to substitute for an object or idea. It is a higher level of thinking that begins in adolescence but does not fully mature until adulthood. 2. Incorrect: Egocentric thinking is where the child thinks the world revolves around them. "It's all about me, me, me!" 3. Incorrect: Animism is the child's way of thinking in which they believe that inanimate objects have feelings, thoughts, and abilities like living things. They think that non-living things behave just like humans.

The family of a client being treated for bleeding esophageal varices asks the nurse why the client is receiving octreotide. How should the nurse respond? 1. "Octreotide is an antibiotic given to decrease the risk of developing an infection." 2. "Taking this medication forms a protective barrier over the varices to prevent bleeding recurrence." 3. "Octreotide helps eliminate ammonia from the body." 4. "This medication lowers the pressure in the liver, so bleeding stops."

4. Correct: Octreotide is a synthetic hormone that selectively inhibits the release of vasodilating hormones in the internal organs. By doing this it decreases blood flow to the liver. When you decrease blood flow to the liver, the pressure in the liver lowers. Less volume, less pressure. So, bleeding should stop. 1. Incorrect:Octreotide is not an antibiotic. 2. Incorrect: You might be thinking of sucralfate, which forms a barrier over an ulcer so acid can't get on the ulcer. 3. Incorrect: No, lactulose decreases ammonia.

A client arrives at the clinic with reports of persistent vomiting, weakness and leg cramps. The nurse notes that the client is irritable. BP 102/58, HR 108, RR 14. Based on this data, what acid/base imbalance does the nurse expect? 1. Respiratory acidosis 2. Respiratory alkalosis 3. Metabolic acidosis 4. Metabolic alkalosis

4. Correct: Symptoms of alkalosis are often due to associated potassium loss and may include irritability, weakness, and cramping. Excessive vomiting eliminates gastric acid and potassium, leading to metabolic alkalosis. 1. Incorrect: Respiratory Acidosis signs and symptoms include decreased respiratory rate, hyportension and a decrease in level of consciousness. Remember, if it's respiratory acidosis, it traces back to the lungs. This problem describes a metabolic issue. 2. Incorrect: Respiratory Alkalosis signs and symptoms include an inability to concentrate, light-headedness, numbness and tingling, tinnitus and loss of consciousness. The loss of CO2 from the lungs would be the problem with respiratory alkalosis, but the problem described in the question is metabolic. 3. Incorrect: Metabolic Acidosis signs and symptoms include headache, confusion, increased respiratory rate and depth, drowsiness, and nausea and vomiting. This can occur in cases of diarrhea, when more bicarb is lost through the lower GI tract.

The nurse is assessing a client one hour post coronary artery bypass graft surgery (CABG). Based on the assessment data, what action should the nurse take? Client increasingly more difficult to arouse. Skin cool/damp. Distended neck veins. Lungs clear bilaterally. Heart sounds distant. CVP 8 mm Hg. BP 90/60. 1. Administer stat dose of clopidogrel. 2. Notify cath lab to prepare for angioplasty. 3. Set up for a central catheter line. 4. Prepare for immediate pericardiocentesis.

4. Correct: The assessment findings point to cardiac tamponade, which is an emergency situation. Did you pick up on the classic s/s of this? Here we see the decreasing level of consciousness and evidence of poor perfusion from decreased cardiac output, distended neck veins from the backward pressure, muffled heart sounds from the fluid collection around the heart, increasing CVP, and the narrowing pulse pressure as the heart is being compressed. Treatment involves a pericardiocentesis to remove blood that has formed around the heart. The primary healthcare provider will insert a needle into the pericardial space to remove the fluid. 1. Incorrect: Clopidogrel is an anti-platelet medication that will not correct cardiac tamponade. 2. Incorrect: If the client were re-occluding, then the client would go to the cath lab or back to surgery. This is not the problem indicated by the signs/symptoms. 3. Incorrect: A central line is not going to correct cardiac tamponade. Immediate removal of the fluid compressing the heart is needed.

During a clinic visit 3 months following a client's diagnosis of type 2 diabetes, the client reports following a 1200 calorie diet and did not bring their glucose-monitoring record. The nurse will anticipate the prescription of which laboratory test? 1. Fasting blood glucose test 2. Urine glucose test 3. Glucose tolerance test (GTT) 4. Glycosylated hemoglobin level (HbA1C)

4. Correct: The glycosylated hemoglobin (Hb A1C) test identifies the average serum glucose attached to hemoglobin over 90 days. The 90 days is correlated with 90 day life of hemoglobin. This test is reflective of how well the client's diabetes is controlled. The client has no restrictions prior to the test. 1. Incorrect: A fasting blood level indicates only the glucose level for at least 8 hours. Urine glucose testing is not an accurate reflection of blood glucose level and does not reflect the glucose over a prolonged time. 2. Incorrect: Urine glucose testing is not an accurate reflection of blood glucose level and does not identify the average glucose level over a prolonged time. The test will identify whether there is an elevated amount of glucose in the urine. 3. Incorrect: Glucose tolerance requires the client fast for the first serum sample and then drink a glucose drink with serum samples taken at specified times. This client has not fasted prior to the office visit. This test is not appropriate at this time.

The nurse provided a community safety presentation for parents and included car seat safety. Which would demonstrate to the nurse that the parents correctly understood the teaching for a 2 month old weighing 10 pounds (4.55 kg)? 1. The car seat is placed upright in the rear facing position in the front passenger seat. 2. Padding is placed under the young infant's head in the semi-reclined car seat in a rear facing position. 3. The car seat is secured in the side of the rear seat in a reclined, front-facing position. 4. The car seat is placed semi-reclined in the middle of the back seat in a rear-facing position.

4. Correct: The guideline for infants < 20 pounds (9kg) is to place them in the middle of the back seat in a rear-facing, semi-reclined car seat. This provides the best protection for their heavy head and weak neck. 1. Incorrect: Infants and young children should never be placed in the front passenger seat, regardless of the direction that the car seat is facing. 2. Incorrect: Padding should not be placed under or behind an infant or child in the car seat because this could become compressed during a crash and cause slackness in the car seat harness. The infant or child could then be ejected from the car seat. 3. Incorrect: Although the car seat is in the back seat, it should be rear-facing, not front-facing, for this 2 month old infant. Also, the middle of the back seat is preferable for car seat placement.

The nurse is caring for a client post cardiac catheterization that was performed via the right femoral artery. What assessment finding in the right lower extremity would be of concern to the nurse? 1. Right pedal pulse 2+/4+. 2. Capillary refill 2 seconds. 3. Erythema. 4. Slight oozing of blood.

4. Correct: The number 1 complication is bleeding. So slight oozing of blood is a problem. Assume the Worse! This is bleeding and you must do something. 1. Incorrect: 2+/4+ is a normal pulse amplitude. We worry about 1+. 2. Incorrect: This is a normal capillary refill. Remember, we want color to return within 2 seconds. 3. Incorrect: Erythema is redness of the skin or mucous membranes, caused by an increased blood flow in superficial capillaries. We are worried about decreased blood flow which would be evidenced by pallor.

The nurse has presented information regarding true versus false labor to a woman in her third trimester of pregnancy. Which statement by the woman would indicate to the nurse that the client understands the information provided? 1. "With false labor the discomfort starts in the back and radiates to the abdomen." 2. "I will experience irregular contractions with both true and false labor." 3. "Contractions during true labor will increase in duration but will decrease in frequency." 4. "Pain increases with a change in activity if I am having true labor."

4. Correct: The pain level will increase with a change in activity if the woman is in true labor. It decreases or goes away if it is false labor. 1. Incorrect: With true labor the discomfort is often in the back and radiates all the way around to the abdomen. False labor discomfort is just in the abdomen. 2. Incorrect: Contractions will be regular with true labor and irregular with false labor. 3. Incorrect: Contractions during true labor will increase in duration and in frequency.

Which initial arterial blood gas (ABG) results would the nurse likely see in a client who has overdosed on acetylsalicylic acid (ASA)? 1. pH 7.50, PaCO2 42, PaO2 63, SaO2 91, HCO3 28 2. pH 7.32, PaCO2 36, PaO2 83, SaO2 95, HCO3 19 3. pH 7.28, PaCO2 28, PaO2 72, SaO2 90, HCO3 16 4. pH 7.48, PaCO2 30, PaO2 88, SaO2 92, HCO3 24

4. Correct: This ABG result indicates respiratory alkalosis. Initially, acetylsalicylic acid stimulates the respiratory center and causes an increase in respiratory rate and depth. This causes respiratory alkalosis by blowing off CO2 and causing the pH to increase. Losing CO2 (acid) makes the client more alkalotic, which is reflected with an increased pH, decreased PaCO​2 and normal HCO​3. 1. Incorrect: This ABG result indicates metabolic alkalosis. The pH is high, PaCO​2 is normal and HCO​3 is high. Normal pH is 7.35-7.45, normal PaCO​2 is 35-45, normal HCO​3 is 22-26. 2. Incorrect: The client with an initial aspirin overdose will have a respiratory alkalosis. This ABG result indicates metabolic acidosis. The pH is less than 7.35 (acidotic); the PaCO2 is within normal limits, and the bicarb is low (less than 22), which creates acidosis. 3. Incorrect: This ABG indicates partially compensated metabolic acidosis. The problem in the stem would initially result in a respiratory problem. The pH is low (acidosis). The PaCO2 is low (alkalosis) as the body tries to compensate by decreasing the acid in the body. The metabolic chemical, bicarb, is low (acidosis) which matches the acidotic pH. Since the pH, PaCO2, and bicarb are all abnormal, we know that partial compensation has occurred.

What does the nurse need to remember when caring for clients on the oncology unit who have a radiation implant? 1. Nursing assignments should be rotated weekly. 2. The nurse should care for no more than 3 clients with a radiation implant per shift. 3. Limit visitors to 60 minutes per day. 4. Wear film badge throughout assigned shift. 5. Educate visitors to stay at least 6 feet from the client.

4., & 5. Correct. Wear a film badge at all times so that you know how much radiation you are getting.Visitors should stay at least 6 feet from the source to decrease exposure to radiation. The closer you get the more radiation exposure. 1. Incorrect: Nursing assignments should be rotated daily, so that the nurse is not continuously exposed. 2. Incorrect: The nurse should only care for one client with a radiation implant in a given shift. 3. Incorrect: Visitors should be limited to 30 minutes per day in order to decrease exposure to radiation.

How would the nurse interpret this client's Arterial Blood Gas (ABG) results? pH 7.44 PaCO2 51 mm Hg Bicarb 31 mEq/liter PaO2 91 mm Hg SaO2 91% 1. Respiratory acidosis 2. Respiratory alkalosis 3. Metabolic acidosis 4. Metabolic alkalosis 5. Uncompensated 6. Partially compensated 7. Fully compensated

4., & 7. Correct. The pH is on the alkalosis side of normal (7.35-7.45). Anything above 7.0 is on the alkalotic side of normal. Look at the CO2. The CO2 is high, which indicates acidosis, so this does not match the alkalotic pH, does it? No. Look at the Bicarb. The bicarb is high, indicating alkalosis, so there is your match. The bicarb is higher than 26, so there is a lot of base in the body. So, this is metabolic alkalosis. Has compensation begun? Yes. The PaCO2 is high. The lungs are attempting to compensate by holding on to carbon dioxide, an acid, to make the pH normal. Since the pH is normal, full compensation has occurred. 1. Incorrect. For this problem to indicate respiratory acidosis, the pH would need to be less than 7.35 (or less than 7.40 if fully compensated to be on the acidotic side of normal) and the CO2 would need to be greater than 45. This pH does not indicate acidosis, so the lungs are not the problem. The lungs are compensating for a metabolic problem. 2. Incorrect. This is not a respiratory problem. The lung chemical, carbon dioxide is high, which would cause acidosis. However, this does not match the alkalotic pH. The pH indicates alkalosis, not acidosis. 3. Incorrect. The pH would need to be below 7.35 (or if fully compensated, the ph would be less than 4.0 to be on the acidotic side of normal) and the Bicarb below 22 for the clint to have metabolic acidosis. 5. Incorrect. The pH is normal even though the PaCO2 and the bicarb values are abnormal, so compensation has occurred. The pH would be abnormal and the PaCO2 would be normal if compensation had not begun. This client has fully compensated. 6. Incorrect. The pH is normal even though the PaCO2 and the bicarb values are abnormal so compensation has occurred. With partial compensation, the pH, PaCO2, and bicarb would all be abnormal. This client has fully compensated.

A client arrives at the emergency department after sustaining full thickness burns. What does the nurse estimate the total body surface area (TBSA) burned to be when using the rule of nines?

Posterior trunk = 18 Posterior arm = 4.5 TBSA burned = 22.5%

The nurse performs an initial assessment on a client admitted following a motor vehicle crash. Based on this assessment, what Glasgow Coma Scale (GSC) score would the nurse assign to the client? Client obtunded, with occassional moaning noted. Opened eyes and extended arm during IV start.

Remember, that the GCS looks at eye opening, motor response and verbal response. So the client opened eyes because of pain with the IV start so that is a 2 for eye opening. During the painful experience, the client extended the arm which is a 2 for motor response. The client is obtunded an occasionally moans, so verbal response is a 2. So the GCS score is 6.

A client weighing 166 pounds (75 kg) is brought to the emergency room with burns to the front and back of both legs and feet. Using the American Burn Association formula to calculate the amount of fluid needed for the first 24 hours, the nurse should set the infusion rate at what for the first eight hours? (Round to nearest whole number).

The American Burn Association formula is 2 - 4mL x weight in kilograms x total surface area burned. Based on the Rule of Nines for adults, a leg is 9% on the front and 9% on the back, which includes the feet. So both legs equal 36% (9% times 4) total surface area burned. The standard multiplier for thermal burns is considered to be 2 mL. Therefore: 2mL x 75 kg x 36 = 5,400 mL for 24 hours. Half that amount, or 2700 mL, should be infused in the first eight hours. Dividing that amount by 8 hours, the infusion rate would be 338 mL per hour.


Set pelajaran terkait

PSYCH-Abnormal Behavior in Children- 2

View Set

Principles of Business Management ch 6-10

View Set

CIT 1351: Chapter 3- System Configuration

View Set

Home Sweet Home - Training & Getting Started

View Set